Blepharoplasty 01-22 Flashcards

1
Q

A patient who is immediately postoperative forehead feminization as well as transconjunctival lower blepharoplasty complains of pain in the right eye and photophobia. On examination, she has marked swelling of both upper eyelids and increased epiphora and chemosis on the right. Which of the following is the most appropriate next step in management?

A) Administer topical anesthetic eye drops
B) Increase intravenous pain medication and serial examinations
C) Perform emergent bedside canthotomy and cantholysis
D) Perform emergent bedside canthotomy only
E) Return to the operating room for examination under anesthesia

A

The correct response is Option A.

The most likely diagnosis in this patient is a corneal abrasion. The cornea is highly innervated (300 to 400 times greater than skin) and any abrasion is associated with marked pain and epiphora. Topical anesthetic eye drops, such as proparacaine hydrochloride or tetracaine hydrochloride, can both treat the pain and be diagnostic. In the setting of a corneal abrasion, the patient would get relief within minutes after application and a formal ocular examination would then be possible. Treatment for a corneal abrasion would include symptom control with topical anesthetics as needed and prophylactic topical antibiotics. Eye patches should be avoided since they can decrease oxygenation of the cornea. Similarly, contact lenses should be omitted until the cornea is fully healed. The cause of corneal abrasions is any mechanical injury during induction, preparation, surgery, and emergence. Particular care should be taken by all team members during the entire surgery and up until the moment when the patient is coordinated enough to touch their face. A time that is particularly dangerous is the cleaning of the face at the end of surgery.

Camouflaging an acute pain episode with narcotics, without establishing and treating the root cause, is not sound medical care and can lead to a missed diagnosis with associated harm.

A return to the operating room, as a first step in management, is not indicated in this clinical scenario. If possible, an ophthalmological examination is best performed with an awake patient.

Both forehead feminization and lower blepharoplasty can lead to a retrobulbar hematoma, which may present similar to a corneal abrasion. Physical examination can distinguish between the two entities since a retrobulbar hematoma would present with proptosis. However, a canthotomy involves only the release of the lateral canthus without severance of the lateral canthal tendon that is found more posterior. This incision would inadequately treat a retrobulbar hematoma.

If topical anesthetic eye drops do not have an effect, then further escalation of care is warranted. If an acute retrobulbar hematoma is suspected, a canthotomy and cantholysis can be indicated. This is a procedure that can be done at bedside should the patient be unable to immediately return to the operating room. It is important that this is done properly and that the lateral canthal tendon is entirely released so that the retrobulbar pressure is released.

How well did you know this?
1
Not at all
2
3
4
5
Perfectly
2
Q

Which of the following surgical site preparation products is most likely to cause significant eye injury and should be avoided for use in or around the eye?

A) 3% Chloroxylenol
B) 4% Chlorhexidine
C) 5% Povidone-iodine
D) Unscented baby shampoo with sterile water rinse

A

The correct response is Option B.

Chlorhexidine is a known ocular irritant at 4% strength and can cause severe corneal damage and corneal toxicity, especially if inadvertently left to dwell in the eye during a procedure. Thus, products like hexachlorophene and chlorhexidine gluconate should be avoided in and around the eye. Dilute chlorhexidine at 0.04% can be safely used as an eye treatment and is clinically prescribed for acanthamoeba keratitis, associated with contact lens use.

5% Povidone-iodine is available in a sterile ophthalmic variety and is used for ophthalmic procedures. 10% Povidone-iodine may also be used around the cheeks, brows, and eyelids. Unscented baby shampoo with a sterile rinse can be used for those with povidone-iodine allergies. While chloroxylenol is a mild eye irritant, it can be used with care around the eye.

How well did you know this?
1
Not at all
2
3
4
5
Perfectly
3
Q

A patient reports diplopia 6 weeks after transconjunctival lower blepharoplasty. The best way to isolate the function of the muscle that is most likely injured is to have the patient move the eye in which of the following directions?

A) Inferior only
B) Lateral and inferior
C) Lateral and superior
D) Medial and inferior
E) Medial and superior

A

The correct response is Option E.

The inferior oblique muscle is the most commonly injured muscle during lower blepharoplasty.

The extraocular muscles include the lateral rectus, medial rectus, superior rectus, inferior rectus, superior oblique, and inferior oblique. Vertical movements require coordination between the superior and inferior rectus muscles as well as the oblique muscles. The contribution of each muscle group depends on the horizontal position of the eye.

When the eye is adducted, the oblique muscles are the primary vertical movers. The inferior oblique muscle causes elevation, and the superior oblique muscle causes depression.

When the eye is abducted, the rectus muscles are the primary vertical movers. Elevation is due to the superior rectus, and depression is due to the inferior rectus.

When the eye is looking straight, both of these groups contribute to the vertical movements.

How well did you know this?
1
Not at all
2
3
4
5
Perfectly
4
Q

A 45-year-old woman presents to the plastic surgeon’s office with complaints of dryness of the eyes and a burning sensation 1 week after a bilateral upper blepharoplasty with skin and muscle excision only by another surgeon. Her pre-operative history included use of contact lenses for over 20 years and laser-assisted in situ keratomileusis (LASIK) 3 months prior to surgery. Her pre-operative examination showed dermatochalasis of both upper eyelids, positive orbital vector, and normal upper eyelid position, and corneal protectors are used intraoperatively. Which of the following is the most likely cause of her postoperative symptoms?

A) Blepharoplasty technique used
B) History of contact lens use
C) History of laser-assisted in situ keratomileusis (LASIK)
D) Positive vector orbit
E) Use of intraoperative corneal protectors

A

The correct response is Option C.

Blepharoplasty should not be performed before 6 months following LASIK surgery. This delay is necessary to allow sensation to return to the cornea. The dryness is from the lack of sensation and decreased blinking leading to increased evaporation.

Transient dry eyes occur in up to 26% of patients after blepharoplasty. Initial treatment of dry eye consists of ocular lubrication. Patients with proptosis, exophthalmos, horizontal lid laxity, or a negative vector orbit are more prone to dry-eye syndrome.

Patients who wear contact lenses without difficulty have adequate tear production. If patients have been advised to not wear contact lenses, they should be evaluated for inadequate tear production.

Patients with prominent eyes have a negative vector orbit and are at increased risk of dry eyes. Positive vector indicates a posterior relationship of the anterior cornea to the malar eminence and does not increase the risk of dry eyes.

Blepharoplasty, upper or lower, will increase the risk of developing dry eye postoperatively. The risk is unrelated to technique and upper blepharoplasty may have less risk compared to lower blepharoplasty. When patients have risk factors for dry eyes postoperatively and want upper and lower blepharoplasty, consider staging the upper and lower blepharoplasties.

Intraoperative use of corneal protectors and lubricant is protective because it decreases ocular exposure.

How well did you know this?
1
Not at all
2
3
4
5
Perfectly
5
Q

Which of the following locations contains the extraocular muscle that is most likely to be injured during transconjunctival lower eyelid blepharoplasty?

A) Between the central and lateral compartments
B) Between the nasal and central compartments
C) Medial to lateral compartment
D) Medial to nasal compartment
E) Superior to central compartment

A

The correct response is Option B.

The inferior oblique muscle is the most commonly injured extraocular muscle during lower blepharoplasty and lies between the nasal and central fat pads. Injury can lead to diplopia typically seen postoperatively between 1 and 6 weeks. Initial treatment is conservative. Surgical repair can be attempted only after improvement stops.

Inferior oblique muscle is not found in the other locations.

The arcuate expansion of Lockwood ligament is located between the central and lateral fat pads.

How well did you know this?
1
Not at all
2
3
4
5
Perfectly
6
Q

A 55-year-old man presents for correction of periorbital puffiness under the eyes with a tear-trough deformity. Transconjunctival lower lid blepharoplasty, with release of the tear trough ligament, and fat redistribution are planned. Which of the following anatomical landmarks indicates entrance into the premaxillary space and release of the tear-trough?

A) Levator labii superioris
B) Levator labii superioris alaeque nasi
C) Orbicularis oculi
D) Zygomaticus major
E) Zygomaticus minor

A

The correct response is Option A.

Through a transconjunctival approach, the dissection begins in a preseptal plane and is then converted into a supraperiosteal dissection over the anterior orbital rim until the levator labii superioris is visualized. This ensures release of the tear-trough ligament and entrance in the premaxillary plane. Orbicularis oculi is the muscle that is released and is found more superficial, as are the zygomaticus major and minor, and levator labii superioris alaeque nasi.

How well did you know this?
1
Not at all
2
3
4
5
Perfectly
7
Q

A 60-year-old woman presents to the emergency department with severe pain and pressure in her right eye. She is 12 hours postoperative from a transcutaneous lower blepharoplasty performed elsewhere. Her pain and pressure began very early postoperatively. Although she called the clinic to report her symptoms, she was told to “take more pain medication.” On physical examination, there is bruising around the right eye with proptosis, diminished range of motion of the eye with diplopia, and decreased visual acuity compared with the left eye. Which of the following is the most likely diagnosis?

A) Acute glaucoma
B) Extraocular muscle entrapment
C) Orbital cellulitis
D) Retrobulbar hematoma
E) Superior orbital fissure syndrome

A

The correct response is Option D.

This patient is presenting with signs and symptoms compatible with retrobulbar hemorrhage following her blepharoplasty procedure. This is an emergency requiring prompt treatment. Symptoms and signs include: severe pain and pressure, decreased visual acuity, and decreased range of motion of her extraocular muscles, which may cause diplopia and nausea. She has marked bruising and proptosis.

Retrobulbar hematoma is the most common cause of visual loss after blepharoplasty. Other eye emergencies, such as extraocular muscle entrapment and superior orbital fissure syndrome, typically occur in the setting of trauma, with orbital bony fractures. While acute glaucoma can present with a painful red eye with diminished visual acuity, especially in elderly patients, it is not typically seen in the early postoperative period following blepharoplasty. Orbital cellulitis is an urgent soft-tissue infection that usually presents with fever, as well as a painful, swollen eye. It has been rarely reported after blepharoplasty but presents about 4 to 6 days following the procedure.

Nahai and associates estimate the risk of visual loss following blepharoplasty at 1:20,000, based on a survey of members of American and British aesthetic surgeons. They found that 82% of patients developed symptoms within the first 24 hours, with more than half of these occurring in the first 6 hours postoperatively. Hypertension and use of aspirin were the two most common comorbidities. When a retrobulbar hemorrhage is suspected, suture removal and even lateral canthotomy should be performed at the bedside while the patient awaits an operating room.

Parallel to surgical measures, medical treatment can be commenced to lower intraocular pressure. Acetazolamide (500 mg intravenously), 20% mannitol (1.5 to 2 g/kg; 12.5 g over 3 min), methylprednisolone (100 mg), betaxolol hydrochloride ophthalmic suspension (one drop then twice daily), and 95% oxygen/5% carbon dioxide can be administered for this purpose. At the same time, an ophthalmology consult should be initiated. Prompt decompression of the orbit is required to avoid permanent visual loss.

How well did you know this?
1
Not at all
2
3
4
5
Perfectly
8
Q

A 62-year-old woman with no medical or ophthalmologic history is scheduled for cosmetic bilateral upper and lower eyelid blepharoplasty under local anesthesia (1% lidocaine with 1/100,000 epinephrine) with intravenous sedation. The procedure is uneventful. At the end of the procedure, the patient develops new-onset bilateral mild lagophthalmos. Stimulation of which of the following structures is the most likely cause of this lagophthalmos?

A) Corrugator supercilii muscles
B) Levator palpebrae superioris muscle
C) Preseptal orbicularis oculi muscle
D) Superior rectus muscle
E) Superior tarsal muscle

A

The correct response is Option E.

Eyelid muscles are innervated by the facial nerve (cranial nerve VII), the oculomotor nerve (cranial nerve III), and sympathetic nerve fibers.

Sympathetic fibers contribute to upper eyelid retraction by innervation of the superior tarsal muscle, also known as the Müller muscle. Sympathetic fibers also innervate the inferior tarsal muscle, contributing to lower lid retraction. The superior tarsal muscle was stimulated by the epinephrine of the local anesthesia, causing temporary mild upper lid retraction.

The oculomotor nerve (cranial nerve III) innervates the main upper eyelid retractor, the levator palpebrae superioris muscle, via its superior branch. Oculomotor nerve is one of the four cranial nerves that transmit parasympathetic fibers.

The facial nerve (cranial nerve VII) innervates the orbicularis oculi, frontalis, procerus, and corrugator supercilii muscles, and supports eyelid protraction. The temporal and zygomatic branches of the facial nerve supply the orbicularis oculi, the main eyelid protractor. The facial nerve also supplies the corrugator supercilii and the procerus, both of which secondarily contribute to upper eyelid protraction.

How well did you know this?
1
Not at all
2
3
4
5
Perfectly
9
Q

A 56-year-old woman presents for a consultation for upper and lower blepharoplasty. She notes that she has a history of dry eyes. Abnormal production of which of the following is the most likely contributor to this patient’s condition?

A) Lipids
B) Lymph
C) Mucin
D) Protein
E) Sebum

A

The correct response is Option A.

The tear film is comprised of three layers: an outer lipid layer, a middle aqueous layer, and an inner mucin layer. The meibomian glands produce the outer lipid layer, which prevents tears from evaporating. Thus, meibomian gland dysfunction can lead to dry eyes. Anatomically, Meibomian glands appear posterior to the tarsal plate with an opening at the eyelid margin. The aqueous layer lubricates the eye and helps clear debris. The mucin layer, closest to the cornea, both nourishes the cornea and allows smooth distribution of tears.

Although sebum is often confused with meibum, or meibomian gland secretions, it is not the same. Sebum blockage of the meibomian glands can cause meibomian gland dysfunction. Thus, sebum is incorrect because while sebum can block the Meibomian glands, it is specifically the Meibomian gland dysfunction which causes the dry eye.

How well did you know this?
1
Not at all
2
3
4
5
Perfectly
10
Q

A 65-year-old woman is evaluated for inferior scleral show appearing 4 weeks after undergoing lower eyelid blepharoplasty. Preoperative examination shows the presence of a positive lower eyelid vector, horizontal lid laxity, and a retropositioned globe. The patient is euthyroid and is treated for open-angle glaucoma. Which of the following findings is most likely to predispose to the development of post-operative lower lid malposition?

A) Euthyroid status
B) Horizontal lid laxity
C) Open-angle glaucoma
D) Positive lower eyelid vector
E) Retropositioned globe

A

The correct response is Option B.

Complications arising as a result of blepharoplasty can be categorized as early (occurring during the first postoperative week), intermediate (occurring between the first and sixth postoperative weeks), and late (occurring after the sixth postoperative week).

Among the intermediate post-blepharoplasty complications, lower eyelid malposition is the most commonly reported. It results from the imbalance in tension between the anterior and posterior lamellae. Widely regarded predisposing factors include: negative vector in which the orbital rim is retropositioned relative to the vertical plane of the cornea, excessive skin resection, aggressive imbrication of the orbital septum, thyroid ophthalmopathy (e.g. Graves disease) with exophthalmos, excessive and/or persistent edema, and hematoma.

Horizontal laxity of the tarsoligamentous sling is the most likely predictor of lower eyelid malposition after lower eyelid blepharoplasty. It reflects the loss of normal upward, posterior, and superior tension across the lower eyelid margin. When this condition exists, manifested by a “snap test” in which the lid can be manually distracted anteriorly more than 8 mm away from the globe, surgical correction of horizontal eyelid laxity should be addressed during the lower eyelid blepharoplasty. This is accomplished with a tarsal strip procedure or similar form of canthoplasty. The euthyroid status with or without globe retropositioning of this patient confers less risk than hyperfunctioning thyroid conditions such as Graves in which exophthalmos increases the potential for lower eyelid malposition. A retropositioned globe will not affect the lower lid as much as the upper lid. Medical conditions such as glaucoma do not influence lower eyelid position.

How well did you know this?
1
Not at all
2
3
4
5
Perfectly
11
Q

A 62-year-old man with a history of pseudoherniation of the lower eyelid fat pad undergoes a bilateral blepharoplasty with a transconjunctival approach to manage the fat compartments and the pinch blepharoplasty technique to manage excess lower eyelid skin. During the procedure, the surgeon notes difficulty in locating the medial fat pad on the right side despite aggressive dissection. Three weeks postoperatively, the patient comes for follow-up and continues to have difficulty with elevating, abducting, and extorting the eye. Which of the following muscles was most likely injured during the procedure?

A) Inferior oblique
B) Inferior rectus
C) Pretarsal orbicularis oculi
D) Superior oblique
E) Superior rectus

A

The correct response is Option A.

The inferior oblique muscle is a thin, narrow skeletal muscle near the anterior margin of the floor of the orbit. This extraocular muscle is attached at its origin, maxillary bone, and the posterior, inferior, lateral surface of its insertion, and eye. The inferior oblique muscle receives its innervation by the inferior branch of the oculomotor nerve. This muscle moves the eye by extorsion, elevation, and abduction. The primary action of the inferior oblique muscle is extorsion/external rotation; secondary action is elevation; tertiary action is abduction. This muscle serves as the primary partition between the medial and central fat compartments of the lower eyelid. Careful identification and preservation of the muscle is important during lower eyelid surgery. In this case, the surgeon likely injured the muscle when having difficulty with fat resection of the medial fat pad.

The inferior rectus muscle is responsible for depressing, adducting, and extorting the eye. The orbicularis oculi muscle is responsible for closing the eye. The superior oblique muscle is responsible for intorsion, depression, and abduction. The superior rectus is responsible for elevation, adduction, and intorsion.

How well did you know this?
1
Not at all
2
3
4
5
Perfectly
12
Q

Which of the following is associated with the transconjunctival blepharoplasty with the skin pinch technique, when compared with transcutaneous blepharoplasty with a skin-muscle flap?

A) Decreased incidence of scleral show
B) Easier access to the lateral fat compartment
C) Easier access when performing retinacular suture canthopexy
D) Increased incidence of middle lamellar scarring
E) Ineffective resection of lower eyelid skin

A

The correct response is Option A.

Transconjunctival blepharoplasty with the skin pinch technique has a low rate of scleral show, compared with rates seen in studies of the transcutaneous blepharoplasty using a skin-muscle flap. However, because it uses a transconjunctival approach to the fat compartments of the lower eyelid, access is not improved but can be challenging, particularly for the lateral fat pad. This is considered a disadvantage of the transconjunctival method. Skin resection is believed to be more thorough with the skin pinch method than with the classical skin-muscle flap. The incidence of middle lamellar scarring with the technique is believed to be less, since the orbicularis oculi muscle is not violated. Performance of canthopexy is not changed by the selection of this technique.

How well did you know this?
1
Not at all
2
3
4
5
Perfectly
13
Q

A 74-year-old woman comes to the office to discuss blepharoplasty. She has bilateral dermatochalasis and right lid ptosis secondary to levator dehiscence. She does not have dry eyes and states that her vision is much improved after recent corneal refractive surgery. Which of the following is the minimum amount of time after her corneal refractive surgery that this patient should wait before undergoing blepharoplasty?

A) 3 Months
B) 6 Months
C) 9 Months
D) 12 Months
E) None; this patient is not a candidate for blepharoplasty

A

The correct response is Option B.

The current accepted time frame between corneal refractive surgery and blepharoplasty is a minimum of 6 months. Early post-corneal refractive surgery puts the patient at risk for worsening dry eyes and/or keratopathy.

How well did you know this?
1
Not at all
2
3
4
5
Perfectly
14
Q

In an adult patient, which of the following physical findings is most likely to support a diagnosis of involutional blepharoptosis of the upper eyelids?

A) Excess upper eyelid skin
B) Eyelid margin-to-reflex distance of 4.5 mm
C) Higher than normal tarsal crease
D) Less than 8 mm of levator function
E) Upper eyelid eversion upon downgaze

A

The correct response is Option C.

Blepharoptosis occurs as a result of acquired or congenital causes. Among the former causes is the most common one, involutional (senile), as well as traumatic, mechanical, neurogenic, and myogenic. The latter category includes myogenic or neurogenic causes only. Because surgical correction is the only effective, definitive therapy for acquired ptosis, it is important for the clinician to be able to differentiate between the causes. The involutional variety is due to attrition or dehiscence of the levator aponeurosis from the anterior upper part of the tarsal plate. Diagnosis of ptosis is confirmed when there is less than 2.5 mm of distance between the upper eyelid margin and the papillary light reflex. Physical findings that support the diagnosis of involutional ptosis include a lid drop during downgaze, a higher than normal upper lid crease, visibility of the eye through the thinned upper eyelid, and preservation of good levator excursion (greater than 10 mm). Rating the degree of ptosis is based on the upper eyelid margin to pupillary light reflex distance: greater than 4 mm is considered normal. Excess upper eyelid skin can obstruct visual field; however, it is not indicative of involutional ptosis.

How well did you know this?
1
Not at all
2
3
4
5
Perfectly
15
Q

A 65-year-old woman comes to the office because she is concerned about the appearance of her lower eyelid. A photograph is shown. Which of the following maneuvers is most critical to improve her periorbital appearance?

A) Botulinum toxin type A injections in the “crow’s feet” area
B) Fat injection limited to the lower eyelid
C) Orbital malar ligament release and fat repositioning
D) Skin resection and lateral canthopexy
E) Skin resurfacing using a phenol-croton oil peel

A

The correct response is Option C. The primary factor contributing to the patient’s concern is the relationship of the orbital malar ligament, orbital septum and retroseptal orbital fat. Release of the orbital malar ligament and fat repositioning offers the best option for improvement. The retroseptal fat compartments appear to be full so that fat injection alone would risk creating unwanted prominent fullness in the area. Skin resection should be conservative after fat repositioning to avoid lower lid malposition. Phenol-croton oil peeling is an effective treatment for aging changes but would not correct the anatomical relationships that underlie this patient’s chief concern. Botulinum toxin type A injections would not adequately address this patient’s concerns.

How well did you know this?
1
Not at all
2
3
4
5
Perfectly
16
Q

A 50-year-old woman comes to the office seeking cosmetic improvement of the lower eyelids. Transposition of a pedicled postseptal fat graft from the central compartment is planned through a transcutaneous approach. A preoperative photograph is shown. Which of the following is the most appropriate structure to release for transposition?

A) Capsulopalpebral fascia
B) Inferior oblique tendon
C) Lockwood ligament
D) Lower slip of the lateral canthal tendon
E) Orbicularis retaining ligament

A

The correct response is Option E.

The orbicularis retaining ligament or the orbital malar ligament is a bilaminar membrane that spans from the periosteum of the inferior orbital rim to the fascia on the underside of the orbicularis. During aging, this ligament accentuates the orbital malar depression and restricts the orbital fat from blending with the sub oribularis oculi fat (SOOF). This patient presents with a prominent orbitomalar sulcus and tear trough deformity. Release of the medial portion of the orbicularis oculi muscle and the orbicularis retaining ligament allows fat transposition over the orbital rim, thus softening and improving this deformity. This procedure can be performed through either a transcutaneous or transconjunctival approach.

The capsulopalprebral fascia is a retractor of the lower eyelid. It is incised during a transconjunctival fat excision. It is not incised in a transcutaneous approach.

The inferior oblique tendon of the inferior oblique muscle should be protected during lower eyelid surgery. Lockwood’s ligament is a supportive structure of the globe. The inferior limb of the lateral canthal tendon is not released for fat transposition. It may be released for canthal support and repositioning.

How well did you know this?
1
Not at all
2
3
4
5
Perfectly
17
Q

A 53-year-old woman is evaluated 6 weeks after undergoing blepharoplasty. Physical examination shows 1 mm of lagophthalmos with no dry-eye symptoms. The patient returns 8 months later for evaluation of dry eye, which began one month after undergoing laser-assisted in situ keratomileusis (LASIK) surgery. Which of the following is the most likely reason for this patient’s dry-eye symptoms?

A) Blunted blink reflex secondary to decreased corneal reflex arc
B) Chronic use of vasoconstrictive eyedrops
C) Transient decrease in functioning of the orbicular muscle of the eye secondary to stretching from lid traction during surgery
D) Transient decrease in tear production caused by lacrimal gland pressure injury
E) Transient hypersensitivity of the cornea

A

The correct response is Option A.

Minimal lagophthalmos in the postoperative period following blepharoplasty is not uncommon and generally self-correcting. Minimal lagophthalmos may persist but is often asymptomatic owing to compensatory blinking and increased tear production, both of which are the result of the mild exposure and resulting corneal stimulation.

Laser-assisted in situ keratomileusis (LASIK) procedure involves creation of a corneal flap that interrupts the long ciliary nerves of the ophthalmic division of the trigeminal nerve. The interruption of these nerves results in decreased sensation to the cornea and a decreased corneal reflex arc. Patients with compensated exposure from blepharoplasty may lose the compensatory blink in reaction to corneal irritation following LASIK. This may cause dry-eye symptoms. With time, the corneal reflex arc improves, and the transient neurotrophic keratopathy and dry-eye symptoms improve. The other options are unlikely to cause her symptoms of dry eyes.

How well did you know this?
1
Not at all
2
3
4
5
Perfectly
18
Q

A 45-year-old woman comes to the office to discuss aesthetic improvement of the lower eyelids. Physical examination shows lower eyelid pseudoherniation of fat and fine skin wrinkles. A postseptal transconjunctival approach with skin pinch excision is considered. The most significant advantage of this approach compared with a transcutaneous skin-muscle flap is a lower risk of which of the following complications?

A) Capsulopalpebral fascia injury
B) Corneal injury
C) Hematoma
D) Infection
E) Lid malposition

A

The correct response is Option E.

Transconjunctival blepharoplasty preserves the middle lamella, which includes the orbicularis oculi muscle. Preserving this layer significantly decreases the incidence of ectropion and lower eyelid malposition. Disadvantages of this technique include more difficulty with visualization and access. Many authors believe that a cutaneous skin muscle approach is more effective in blending the lid-cheek junction and transposing fat.

Some studies have shown minor reductions in hematoma and infection rates with the transconjunctival approach; however, the differences are small and not the most significant advantage.

The potential for corneal injury is greater with the transconjunctival approach. Most surgeons use corneal shields to prevent this complication.

The capsule palpebral fascia is routinely cut to access the fat compartments during a transconjunctival blepharoplasty.

How well did you know this?
1
Not at all
2
3
4
5
Perfectly
19
Q

When skin excisions are designed for upper blepharoplasty, carrying the medial extent past the punctum is most likely to result in which of the following adverse effects?

A) Inadequate vertical skin excision
B) jury to the lacrimal system
C) Lagophthalmos
D) Retrobulbar hematoma
E) Webbing of the nasal skin

A

The correct response is Option E.

Extending the skin incisions too far medially can create a webbing of the nasal skin. Carrying the incision medially does not promote inadequate skin excision, lagophthalmos, or retrobulbar hematoma. Injury to the lacrimal system is unlikely to occur with a more medial incision.

How well did you know this?
1
Not at all
2
3
4
5
Perfectly
20
Q

A 61-year-old woman is evaluated 7 hours after undergoing bilateral lower blepharoplasty with septal release and fat repositioning. She reports pain and diminished vision in the right eye. Physical examination shows more swelling on the right side than on the left and tender proptosis of the right eye. Which of the following is the most appropriate treatment?

A) Administration of stronger pain medication
B) Head elevation and ice packs
C) Lasix administration and observation
D) Orbital decompression
E) Tarsorrhaphy

A

The correct response is Option D.

The most appropriate treatment is orbital decompression for retrobulbar hematoma. Most complications after blepharoplasty are correctable and minor. However, postoperative bleeding into the orbit causes a retrobulbar hematoma. With a large enough hematoma, orbital pressure increases, causing increased swelling, eye pain, and proptosis. With continued pressure, the optic nerve is compressed, leading to reduction in vision and ultimately blindness. The cause of a retrobulbar hematoma is bleeding within the orbit. As such, the blepharoplasty technique would be one in which the orbital septum was opened. Nevertheless, this complication must be considered in all blepharoplasty patients.

The treatment for retrobulbar hematoma is emergent orbital decompression. The incision needs to be opened and the orbital space accessed for the hematoma and soft tissues to decompress and alleviate traction or pressure on the optic nerve. A lateral canthotomy is done as well to ensure maximal decompression. In the past, mannitol has been given as a diuretic for reduction of edema, but also as a free radical scavenger in hopes of protecting the optic nerve from compressive ischemia. Lasix is often administered as a quick controllable diuretic to lessen edema.

This is a very rare complication of blepharoplasty. Mejia et al. surveyed plastic surgeons in the United States and Great Britain and derived data from over 750,000 blepharoplasties. In this study, 25 patients had permanent visual loss and 14 had temporary loss of vision. The overall incidence of visual loss due to retrobulbar hematoma was 0.0052% or one in 20,000 cases, with permanent loss occurring in 0.0033% or one in 30,000. Symptoms as noted above occurred in all patients within the first 24 hours, and hypertension was found to be the most common risk factor.

All patients will develop postoperative swelling and ecchymosis to varying degrees. Head elevation and ice packs are appropriate for standard swelling noted without findings of orbital pain, proptosis, or changes in vision. Tarsorrhaphy is useful for patients with lagophthalmos or ectropion but has no role in globe protection for acute proptosis in the above setting. Lasix infusion is part of the treatment for retrobulbar hematoma, but as a stand-alone therapy it is inadequate. Provision of more pain medication is inappropriate in the presence of pain and reduced vision and will only serve to mask some symptoms related to the hematoma.

How well did you know this?
1
Not at all
2
3
4
5
Perfectly
21
Q

A 62-year-old woman is evaluated for lower blepharoplasty. On examination, negative vector is noted. Postoperatively, this patient is at increased risk for which of the following conditions?

A) Dystopia
B) Ectropion
C) Enophthalmos
D) Lagophthalmos
E) Proptosis

A

The correct response is Option B.

The finding of a negative vector places the patient at an elevated risk for lower lid malposition and ectropion. The negative vector refers to the anatomic relationship on lateral view of the maximum projecting point of the globe and the maximum projecting point of the infraorbital malar prominence. If the globe projects less than the malar prominence, a negative vector exists. Conversely, if the malar prominence projects more than the globe, a positive vector exists.

The negative vector finding indicates potentially deficient globe and lid support based on skeletal anatomy. Such patients will often have minor scleral show or lateral lid lag. It is important to recognize these findings prior to blepharoplasty surgery in order to surgically address the risks of ectropion via primary lid suspension during the blepharoplasty. Occasionally, lower lid blepharoplasty may be avoided if a negative vector is present and other conditions such as dry eye exist. Other findings or conditions that are associated with postoperative ectropion and lower lid malposition are: orbicular weakness, anterior lamellar shortage, inferior eyelid/orbital volume deficit, and eyelid laxity. Excessive or prominent middle lamellar scarring can occur after surgery, which can also lead to lid malposition.

Lagophthalmos is the inability to lower the upper lid fully and is a negative consequence of upper blepharoplasty due to excessive tissue resection or fibrosis. Enophthalmos is interior retraction or displacement of the globe related to increased orbital volume. This is unrelated to lower blepharoplasty surgery. Proptosis is an external displacement of the globe giving the appearance that the eyeball is extruding from the obit. This is most often associated with Graves disease, head trauma, and increased intracranial pressure. It can also be due to a retrobulbar hematoma after blepharoplasty, which is a surgical emergency due to the risk of blindness.

Dystopia refers to malposition of the globe related to skeletal changes of the orbit. This would not be a result of blepharoplasty, but can occur after facial trauma or facial tumor resection.

How well did you know this?
1
Not at all
2
3
4
5
Perfectly
22
Q

A 60-year-old man comes to the office because he desires improvement in the appearance of his lower eyelid and upper cheek area. Physical examination shows a prominent tear trough. Which of the following is the anatomic basis of the tear trough?

A) Attachment of the orbital septum to the arcus marginalis
B) Cleft between the palpebral and orbital parts of the orbicularis oculi
C) Osteocutaneous ligament arising from the medial portion of the maxilla
D) Prominence of the orbital rim following descent of the malar fat pad
E) Triangular confluence of the origins of the orbicularis oculi, levator labii superioris alaeque nasi, and levator labii superioris

A

The correct response is Option C.

Recent articles have greatly improved our understanding of the anatomy of the lower eyelid, tear trough, pre-zygomatic space, and the ligaments in the orbital area.

The tear trough ligament is a true osteocutaneous ligament between the palpebral and orbital portions of the muscle. It extends inferolaterally from the medial canthus to approximately the mid-pupillary line, where is connects with the bilayered orbicularis retaining ligament. Recent anatomic dissection work has shown that this ligament is the anatomic basis of the tear trough deformity.

Fillers should be placed inferior to the tear trough ligament; placing them superiorly will only serve to emphasize lower eyelid fat, and will emphasize the tear trough deformity.

How well did you know this?
1
Not at all
2
3
4
5
Perfectly
23
Q

A 16-year-old girl comes to the clinic because she is dissatisfied with the asymmetric appearance of her breasts. Physical examination shows the right nipple-areola complex is more superiorly located and the breast volume is small. There is absence of the right anterior axillary fold. Which of the following other physical examination findings is most likely?

A) Accessory nipple
B) Microtia
C) Right clubfoot
D) Scars consistent with repaired cleft lip
E) Shortened right-hand digits

A

The correct response is Option E.

The chest findings described are consistent with Poland syndrome with absence of the sternocostal head of the pectoralis major muscle. Poland syndrome can be associated with hand abnormalities, including shortened digits. Cleft lip, accessory nipple, clubfoot, and microtia are not known to be associated with Poland syndrome.

How well did you know this?
1
Not at all
2
3
4
5
Perfectly
24
Q

A 32-year-old Korean woman undergoes bilateral upper lid blepharoplasty for creation of a supratarsal crease. A partial incision technique is used. Which of the following is the most likely complication of this procedure?

A) Asymmetry
B) Epicanthal webbing
C) Fold loss
D) Lagophthalmos
E) Suture extrusion

A

The correct response is Option A.

Blepharoplasty is the most common facial cosmetic procedure performed on people of Asian descent. Unlike blepharoplasty in Caucasian faces, the goal of Asian blepharoplasty is to create a supratarsal fold. Asian eyelids are characterized by several key elements including absent or low lid crease, shorter tarsus, descending pre-aponeurotic fat, and minimal or absent connection between the levator aponeurosis and the upper lid dermis. The mainstay of surgical correction is creation of a permanent fixation point between the levator muscle and the supratarsal dermis and subdermal structures.

The most common complication after Asian blepharoplasty is asymmetry. It is important to remember that unlike Caucasian blepharoplasty, the motivation for Asian blepharoplasty is less frequently rejuvenation. Rather, Asian patients are typically younger and desire creation of a supratarsal fold or correction of a preexisting fold asymmetry. Asymmetry is a common preoperative finding and should be well documented and discussed with the patient before surgery. Small differences in positioning of the newly created crease can be very obvious to both patient and surgeon. When recognized immediately after surgery, early revision should be considered.

Other complications unique to Asian blepharoplasty include fold loss, suture extrusion, and epicanthal scarring. Lagophthalmos, which most often results from overresection of upper eyelid skin, is less frequent in Asian blepharoplasty because skin excision is typically more limited, particularly in a younger patient undergoing a partial incision technique. Fold loss may result from a technical error in securing the layers of dermis, epidermis, or levator aponeurosis, or from placing too few sutures. Revisional surgery would be required to correct this problem. Suture extrusion is not uncommon, given that permanent sutures are used for fixation. Meticulous placement of clear 7-0 nylon and trimming of suture ends will minimize the risk of this complication. If a suture becomes exposed in the first few months postoperatively, removal should be deferred until scarring is mature and fixation is more tenacious. Epicanthoplasty is commonly used is Asian patients with epicanthal folds undergoing blepharoplasty. Various techniques, including Y, W, and modified Z-plasties, have been reported. The epicanthal region is prone to hypertrophic scarring, particularly in Asians who have thick dermis.

How well did you know this?
1
Not at all
2
3
4
5
Perfectly
25
Q

Which of the following is the function of secretions of the meibomian glands?

A) Coats cornea as inner layer of tear film
B) Lubricates eyelid skin
C) Prevents evaporation of tear film
D) Promotes control of infectious agents
E) Promotes dispersion of tear film

A

The correct response is Option C.

Tears are a trilaminar fluid. The precorneal layer is formed by mucin-secreting goblet cells in the conjunctiva. This inner layer of the tear film covers the cornea and promotes the dispersion of the overlying aqueous layer.

The lacrimal gland secretes the middle layer. This aqueous layer is made of water and proteins. This layer promotes osmotic regulation and the control of infectious agents.

The meibomian glands produce the outer lipid layer. This oil layer helps to prevent the evaporation of the tear film. As a result, dysfunction of the meibomian glands can lead to dry eyes.

How well did you know this?
1
Not at all
2
3
4
5
Perfectly
26
Q

In a patient with facial proportions within the normal ranges, which of the following measurements best approximates intercanthal distance?

A) Eyebrow length
B) Nasal bone width
C) Orbital fissure width
D) Stomion-to-menton distance
E) Subnasale-to-stomion distance

A

The correct response is Option C.

Intercanthal distance most closely approximates orbital fissure width. Normal facial values are often described as proportions rather than absolute numbers. Many texts describe normal intercanthal distance as between 30 and 35 mm, but some studies have shown intercanthal distances of up to 40 mm in healthy cohorts. Thus, using the facial features as referents can be helpful. The face is often divided into fifths for analysis of width, and the intercanthal distance represents one fifth, as does the orbital fissure width. The nasal bone width is narrower than the intercanthal distance, and the eyebrow length extends lateral to the lateral canthus, representing greater than a fifth of the facial width. Subnasale, stomion, and menton distances are most often used to calculate facial height proportions. Although there is no reason why, theoretically, a measurement of facial height could not correspond to a measurement of facial width, these values do not.

How well did you know this?
1
Not at all
2
3
4
5
Perfectly
27
Q

A 65-year-old woman comes to the office for follow-up 6 days after undergoing bilateral upper eyelid blepharoplasty and repair of the right levator aponeurosis. Preoperatively, the patient had bilateral levator excursion of 13 mm and 4 mm of ptosis of the right eyelid. No ptosis of the left eyelid was noted. Physical examination today shows 2 mm of ptosis of the left upper eyelid. The right upper eyelid is well positioned. Which of the following is the most likely explanation for these findings?

A) Hering law
B) Horner syndrome
C) Müller maneuver
D) Todd paresis
E) von Graefe sign

A

The correct response is Option A.

Hering law describes equal innervation to the eyelids in that the signal to the levator is the same despite the potential need for each eyelid to work independently. In the scenario described, the patient had obvious ptosis of the right eyelid, and the signal to raise the eyelids was strong. When the right ptosis was corrected, the signal to raise the eyelids decreased, and the more mild ptosis of the left eyelid was uncovered. To help avoid this problem, a Hering test or a patch test can be performed. The Hering test is performed by elevating the ptotic eyelid and observing whether the other eyelid becomes ptotic. A patch test is when the ptotic eyelid is covered for a period of time (usually 15 minutes) and then observed for whether the non-ptotic eye becomes ptotic. The key to both tests is to decrease the excessive signal to raise the eyelids.

Horner syndrome includes ptosis of the eyelid, constriction of the pupil, and decreased sweating due to disease in the sympathetic system. This can be due to a tumor, congenital or iatrogenic.

von Graefe sign is lagophthalmos in downgaze. This is related to Graves disease. Müller maneuver is the reverse of the Valsalva maneuver. After a forced expiration, an attempt at inspiration is made with closed mouth and nose, thereby creating negative pressure in the chest and lungs. This maneuver is used to find weakened areas of the airway.

Todd paresis is focal weakness following a seizure. This can affect eye position.

How well did you know this?
1
Not at all
2
3
4
5
Perfectly
28
Q

A 48-year-old woman comes to the office because of pain and tearing of the right eye 1 week after undergoing upper eyelid blepharoplasty. Fluorescein stain test result is positive for corneal erosion. On physical examination, absence of which of the following is most likely to put this patient at risk for corneal ulceration?

A) Accommodation reflex
B) Bell phenomenon
C) Ocular convergence
D) Oculocardiac bradycardia
E) Pupillary light response

A

The correct response is Option B.

Transient lagophthalmos during sleep is not uncommon following blepharoplasty. During the first few weeks of recovery, it is important to protect the eyes with lubricating drops and ointment. Bell phenomenon, an upward and outer movement of the eye when the eye is closed, is a protective mechanism which keeps the cornea protected behind the upper eyelid. Bell phenomenon is absent in 10 to 15% of the population. Accommodation reflex, oculocardiac reflex, ocular convergence, and pupillary light response do not place the cornea at risk after blepharoplasty.

How well did you know this?
1
Not at all
2
3
4
5
Perfectly
29
Q

A 45-year-old woman comes to the office because of puffiness of both lower eyelids. Physical examination shows little lower lid skin excess, but prominent eyelid fat pads. Lid tone, snap back, and distraction test results show no abnormalities. Lid-cheek junction is smooth. A retroseptal transconjunctival approach of the lower lid is planned. Regarding the operative approach, which of the following statements is most accurate?

A) Fat pad reduction without violating the orbital septum is performed
B) Lower eyelid skin resurfacing with laser or chemical peel should not be performed concurrently with a retroseptal approach
C) The incision should be placed 1 to 2 mm below the tarsal border
D) The inferior oblique muscle will be noted between the central and lateral fat pad
E) The plane of dissection is deep to the orbicularis, but superficial to the orbital septum

A

The correct response is Option A.

The incision for a retroseptal approach is usually placed 4 to 5 mm below the tarsal border, or about 8 mm for the lid margin. The plan of dissection for a retroseptal approach is, by definition, deep to both the orbicularis muscle and septum. Because the fat pads are retroseptal, modification does not require entry through the septum when a retroseptal approach is used.

The preseptal approach is typically chosen for modification of the lid-cheek junction, and facilitates fat pad redistribution and access to the midface. The retroseptal approach is used for reduction of fat pads only. The inferior oblique muscle is located between the central and medial compartments of fat.

Multiple authors have shown that is it indeed safe to perform skin resurfacing with either chemical peel or laser simultaneously with a transconjunctival blepharoplasty.

How well did you know this?
1
Not at all
2
3
4
5
Perfectly
30
Q

A 50-year-old woman undergoes upper and lower eyelid blepharoplasty with local anesthesia and intravenous sedation. The procedure begins with no patient discomfort; however, the patient reports marked pain once removal of the lower lateral fat pad is initiated. Which of the following nerves is the source of pain in this patient?

A) Facial nerve
B) Infraorbital nerve
C) Infratrochlear nerve
D) Lacrimal nerve
E) Zygomaticofacial nerve

A

The correct response is Option E.

The zygomaticofacial nerve provides sensory innervation to the lateral fat pad of the lower eyelid.

Branches of the trigeminal nerve provide sensation to the face. The infraorbital nerve, the second branch of the trigeminal nerve, supplies innervation to the lower eyelid, cheek, and upper lip. The lateral palpebral branch of the lacrimal nerve, a branch of the infraorbital nerve, supplies sensory innervation to the superior lateral portion of the upper eyelid. The infratrochlear nerve provides sensory innervation to the medial aspect of the upper and lower eyelid. The lacrimal nerve provides sensation to the upper eyelid through the first branch of the trigeminal nerve. The facial nerve is a motor nerve to the face and is not responsible for sensation in the face.

How well did you know this?
1
Not at all
2
3
4
5
Perfectly
31
Q

Which of the following locations contains the extraocular muscle that is most likely to be injured during transconjunctival lower eyelid blepharoplasty?

A) Between the central and lateral compartments
B) Between the nasal and central compartments
C) Medial to lateral compartment
D) Medial to nasal compartment
E) Superior to central compartment

A

The correct response is Option B.

The inferior oblique muscle is the most commonly injured extraocular muscle during lower blepharoplasty and lies between the nasal and central fat pads. Injury can lead to diplopia typically seen postoperatively between 1 and 6 weeks. Initial treatment is conservative. Surgical repair can be attempted only after improvement stops.

Inferior oblique muscle is not found in the other locations.

The arcuate expansion of Lockwood ligament is located between the central and lateral fat pads.

How well did you know this?
1
Not at all
2
3
4
5
Perfectly
32
Q

A 50-year-old woman comes to the office for consultation because of upper eyelid dermatochalasis and lower eyelid bags. History includes hypothyroidism and type 2 diabetes mellitus. She underwent laser eye (Lasik) surgery 1 year ago. Medications include estrogen and thyroid hormone replacements as well as metformin. Upper and lower eyelid blepharoplasty is planned. This patient is at greatest risk for which of the following postoperative complications?

A) Bleeding
B) Blindness
C) Chemosis
D) Dry eye syndrome
Eyelid malposition

A

The correct response is Option D.

Dry eye syndrome (dysfunctional tear syndrome) is a problem of tear deficiency and eye discomfort that may result in damage to the cornea. Combining upper and lower eyelid blepharoplasty relative to staged upper and lower eyelid surgery presents a greater risk for dry eye syndrome due to orbicularis oculi dysfunction after surgery. Women on hormone replacement therapy also have a higher risk for developing dry eye syndrome after blepharoplasty. Eye lubrication before and after surgery needs to be strongly considered. Patients with prior laser vision correction should wait at least 6 months before pursuing blepharoplasty because of the effects on corneal sensation and tear production. In this case, the patient’s Lasik procedure 1 year prior would not impair her surgical outcome with upper and lower eyelid blepharoplasty.

Chemosis is characterized by conjunctival swelling and irritation after blepharoplasty surgery, requiring lubrication after surgery. Some surgeons advocate treatment of this condition with steroid drops.

Eyelid malposition is more likely in patients with a negative vector, defined as those with a maxilla that does not project beyond the orbital rim. It is also common in patients who have poor eyelid tone, diagnosed by snap test or evident as ectropion.

Major adverse postoperative events such as bleeding and loss of vision are rare. Bleeding is more risky with uncontrolled hypertension.

Smoking history, diabetes, and hypothyroidism are not directly associated with specific complications of blepharoplasty.

How well did you know this?
1
Not at all
2
3
4
5
Perfectly
33
Q

A 57-year-old woman comes to the office because she is dissatisfied with the appearance of her eyes. She says they appear “small” and “tired.” Physical examination shows dermatochalasis of the upper eyelids, 2 mm of eyelid ptosis, deep transverse rhytides of the forehead, and fine periorbital rhytides. She elevates her eyebrows 3 mm when she opens her eyelids. A skin-only blepharoplasty with formal eyelid ptosis repair is planned. After the procedure, which of the following clinical findings is most likely in this patient?

A) Blepharospasm
B) Brow ptosis
C) Decreased pretarsal show
D) Decreased volumetric convexity
E) Eyelid retraction

A

The correct response is Option B.

A patient who undergoes blepharoplasty and ptosis repair in the context of a compensated brow ptosis is likely to experience worsened brow ptosis after the procedure. Evaluation of the blepharoplasty patient requires careful examination of the entire upper third of the face. Patients may have, in addition to excess upper eyelid skin, an eyelid ptosis. In addition to identifying the ptosis, it is important to recognize compensated brow ptosis. A compensated eyelid ptosis occurs when the patient uses the frontalis muscles to raise the eyebrows, which results in a functional improvement in visual fields. This is most easily identified by having the patient close her eyes, and evaluate the automatic raising of the eyebrow on eyelid opening. In this case, the change in the position of the brow on downward gaze and on frontal gaze indicates a compensated brow ptosis.

After ptosis repair and blepharoplasty, brow ptosis can become more manifest as the need for compensation decreases.

How well did you know this?
1
Not at all
2
3
4
5
Perfectly
34
Q

A 48-year-old man comes to the office for consultation because he is dissatisfied with the appearance of his puffy, swollen eyelids and dry eyes. Physical examination shows mild bilateral proptosis and injected conjunctivae. Visual acuity and visual field testing are both normal. Extraocular motion testing reveals lid lag. Which of the following is the most likely cause of this patient?s condition?

A ) Allergic conjunctivitis
B ) Myasthenia gravis
C ) Optic neuritis
D ) Sjögren syndrome
E ) Thyroid ophthalmopathy

A

The correct response is Option E.

The most appropriate answer is thyroid eye disease, or thyroid ophthalmopathy. This is the most common cause of proptosis and diplopia in adults. It affects women approximately 4 to 6 times more frequently than men. Puffy, swollen eyelids, injected conjunctivae, eyelid lag, and proptosis are common in thyroid ophthalmopathy.

Optic neuritis is typically associated with visual loss, which is not a symptom in this scenario. Similarly, myasthenia gravis is associated with eyelid ptosis, worsening toward the end of the day—which is not a symptom in this scenario. Although both allergic conjunctivitis and Sjögren syndrome could cause conjunctival injection, neither would be associated with proptosis or eyelid lag.

Vision loss, due to compression of the optic nerve, can occur in the most severe cases. Fortunately, this

How well did you know this?
1
Not at all
2
3
4
5
Perfectly
35
Q

A 36-year-old woman is scheduled for lower blepharoplasty and mid face rhytidectomy using a transconjunctival approach. Which of the following structures is released to access the mid face for suborbicular muscle of the eye fat redraping?

A ) Capsulopalpebral fascia
B ) Lockwood ligament
C ) Orbital septum
D ) Orbitomalar ligament
E ) Parotid masseteric fascia

A

The correct response is Option D.

The orbitomalar ligament attaches the orbicular muscle of the eye to the orbital rim. It separates the lower eyelid from the mid face. Release of this structure is required to obtain access to the mid face when approaching it from the lower eyelid.

The capsulopalpebral fascia inserts on the inferior border of the tarsus. It makes up the anterior superior portion of the lower eyelid retractors distal to the Lockwood ligament. The capsulopalpebral fascia is divided during the transconjunctival incision. This affords access to the lower eyelid for the blepharoplasty procedure.

The Lockwood ligament is a fascial thickening that supports the globe. It surrounds the inferior rectus and inferior oblique muscles and fuses with the capsulopalpebral fascia. It is analogous to the Whitnall ligament in the upper eyelid. The orbital septum acts to contain the orbital contents. It attaches inferiorly to the periosteum at the arcus marginalis and superiorly to the eyelid margin. A transconjunctival blepharoplasty can be performed by a pre- or postseptal approach. The parotid masseteric fascia is within the mid face. It is distal to the orbitomalar ligament when approached from the eyelid. It does not need to be released to access the mid face.

How well did you know this?
1
Not at all
2
3
4
5
Perfectly
36
Q

A 48-year-old woman is brought to the emergency department after sustaining facial injuries in a motor vehicle collision. Physical examination shows a 4-cm laceration of the left upper eyelid. The levator palpebrae superioris is transected just superior to the tarsus, exposing underlying structures. Which of the following intact structures is now exposed?

A ) Lacrimal sac
B ) Müller muscle
C ) Orbital septum
D ) Preaponeurotic fat
E ) Retro-orbicularis oculi fat

A

The correct response is Option B.

Knowledge of periorbital anatomy is critical for plastic surgeons who perform eyelid procedures, both cosmetic and reconstructive. The levator complex originates at the orbital apex at the lesser wing of the sphenoid and travels horizontally until it reaches the Whitnall ligaments, where it changes to a more vertical direction before its aponeurosis inserts on the tarsus, orbital septum, and skin. Above the level of the tarsus, the orbital septum lies anterior to the levator, and preaponeurotic fat lies posterior to the orbital septum.

Retro-orbicularis oculi fat lies anterior to the septum and posterior to the orbicularis oculi.

Müller muscle inserts directly on the tarsus and lies just posterior to the levator directly superior to the tarsus.

How well did you know this?
1
Not at all
2
3
4
5
Perfectly
37
Q

A 50-year-old woman is scheduled to undergo transcutaneous blepharoplasty. Fat transposition and canthopexy are planned. The inferior oblique muscle of the eye is most vulnerable to injury when dissecting between which of the following structures?

A) Central fat compartment and lateral fat compartment
B) Lateral fat compartment and orbicularis retaining ligament
C) Medial canthal tendon and medial fat compartment
D) Medial fat compartment and central fat compartment
E) Orbicularis retaining ligament and arcus marginalis

A

The correct response is Option D.

The inferior oblique muscle of the eye can be found between the medial fat compartment and the central fat compartment. When performing either excision or manipulation of the medial and central fat compartments, the inferior oblique muscle is vulnerable to injury. These injuries include resection, cauterization, scarring, hemorrhage, edema, and suture injury while repairing the septum orbitali. Depending upon the extent of injury, the symptoms can be transient or permanent. One study suggested that transection of less than 50% of the muscle will not cause permanent diplopia.

The orbicularis retaining ligament is a structure that originates along the orbital rim and inserts into the overlying orbicularis oculi muscle. Laterally, it contributes to the lateral canthal ligament. Recent studies have demonstrated that the orbicularis retaining ligament is a circumferential structure. The arcus marginalis is the periosteal extension of the septum orbitale as it attaches into the orbital rim. The inferior oblique muscle lies deep to both of these structures.

How well did you know this?
1
Not at all
2
3
4
5
Perfectly
38
Q

A 50-year-old woman is scheduled to undergo transcutaneous lower eyelid blepharoplasty for periorbital aging. Which of the following is the most likely complication 3 months postoperatively?

A) Chemosis
B) Infection
C) Lacrimal system dysfunction
D) Lagophthalmos
E) Lower eyelid malposition

A

The correct response is Option E.

The most common complication following lower blepharoplasty is lower eyelid malposition. Lower eyelid malposition ranges from mild scleral show to severe cicatricial ectropion. Malposition of the lower eyelid results from abnormal downward forces on the eyelid. Excessive scarring, over-resection of skin, imbrication of the orbital septum, orbicularis paralysis, edema, and hematoma can all produce lower eyelid malposition. Predisposing factors for malposition that should be indentified preoperatively include malar hypoplasia, globe proptosis, high myopia, laxity of the lower eyelid, and thyroid ophthalmopathy. Postoperative mild scleral show can often be managed with massage and topical lubrication. However, severe ectropion may require scar release, skin grafting, or a lower eyelid tightening procedure such as a canthoplasty.

Chemosis is a postoperative sequela, but it usually resolves in 6 weeks. Infection of the lower eyelid following blepharoplasty is rare given that the eyelid is well vascularized. Lacrimal system dysfunction is rare and often returns to normal without surgical intervention. Lagophthalmos can occur following upper blepharoplasty and can lead to postoperative pain, guarding, or incomplete eyelid closure. Lagophthalmos is usually temporary and resolves with lubrication and eyelid massage.

How well did you know this?
1
Not at all
2
3
4
5
Perfectly
39
Q

A 69-year-old woman comes to the office for consultation regarding lower eyelid blepharoplasty. Physical examination shows a negative canthal tilt. Snap-back testing of the lower eyelid shows 8 mm of distraction. Transcutaneous lower blepharoplasty with fat transposition is planned. Which of the following is the most appropriate surgical choice to avoid postoperative malposition of the lower eyelid?

A) Horizontal wedge excision
B) Lateral canthoplasty
C) Orbicularis repositioning
D) Posterior lamellar graft
E) Tarsorrhaphy

A

The correct response is Option B.

Lateral canthal support can treat preexisting lower eyelid laxity and can help protect against postblepharoplasty malposition of the lower eyelid. The choice of treatment can be determined by the extent of eyelid laxity. For severe lower eyelid laxity (greater than 6 mm of eyelid distraction), a lateral canthoplasty with lateral cantholysis allows for increased superior mobility and precise positioning of the lower canthal tendon inside the orbital rim. While horizontal wedge excision of the lower eyelid addresses the horizontal lower eyelid laxity, it is not recommended in negative canthal tilt patients (lateral canthus at lower level than medial canthus), who also require lateral eyelid resuspension. Mild eyelid laxity (1 to 2 mm of eyelid distraction) can also be addressed with orbicularis repositioning. A posterior lamellar graft would not be an appropriate treatment for this condition. In minimal cases of eyelid laxity, temporary external support of the lower eyelid with a tarsorrhaphy stitch at the lateral limbus can help protect the cornea and prevent early cicatricial ectropion.

How well did you know this?
1
Not at all
2
3
4
5
Perfectly
40
Q

A 50-year-old woman comes to the office because of numbness of the skin of the right central forehead 10 weeks after undergoing upper eyelid blepharoplasty. The procedure involved transpalpebral resection of the medial brow depressor muscles. Injury to which of the following nerves is the most likely cause of this patient’s condition?

A) Abducens
B) Oculomotor
C) Supraorbital
D) Supratrochlear
E) Zygomaticotemporal

A

The correct response is Option D.

The most likely cause of numbness in the patient described is injury to the supratrochlear nerve. The supratrochlear nerve courses superiorly through the corrugator muscle and innervates the central forehead skin. It is subject to injury during muscle interruption via either of the transpalpebral, coronal, or endoscopic approaches. The abducens nerve is motor to the lateral rectus muscle of the globe and is not within the operative field. Although it can be injured distal to its exit from the frontal bone, the deep branch of the supraorbital nerve provides sensation to the frontal periosteum. The frontal branches of the facial nerve are motor only to the frontalis muscle. The oculomotor nerve would not result in these findings, as it does not supply sensation to the forehead. The zygomaticotemporal nerve renders sensation to the lateral orbital-forehead skin and is located far lateral to the medial brow depressors.

How well did you know this?
1
Not at all
2
3
4
5
Perfectly
41
Q

A 4-year-old boy is referred for evaluation because of a 6-month history of bilateral epiphora. Examination shows that the eyelashes are rubbing against the inferior corneas bilaterally. Which of the following is the most likely cause of this patient’s condition?

A ) Abnormal attachment of the orbital septum

B ) High-riding tarsal plane

C ) Laxity of the lateral canthal tendons

D ) Laxity of the tarsal plate

E ) Redundancy of skin and orbicular muscle of the eye

A

The correct response is Option E.

The most common cause of epiblepharon is excess pretarsal skin and orbicular muscle at the lower eyelid margin. In this congenital anomaly, a fold of skin and underlying orbicular muscle of the eye override the eyelid margin, often pushing the cilia against the globe. The eyelid margin and tarsus are stable and maintain the proper orientation. Epiblepharon usually affects the lower eyelids, is more common in people of Asian descent, and is accentuated on downward gaze.

Laxity of the tarsal plate and the lateral canthal tendons are seen in the aging lower eyelid. Abnormal attachment of the orbital septum can be seen after trauma.

42
Q

A 66-year-old man is referred for evaluation of left eyelid ptosis. He reports decreased vision in the left eye only, which bothers him throughout the day. Physical examination shows mild-to-moderate left eyelid ptosis and elevation of the left supratarsal crease. Brow position and function and pupillary size are normal bilaterally. Which of the following is the most likely diagnosis?

A ) Congenital ptosis

B ) Facial nerve injury

C ) Horner syndrome

D ) Myasthenia gravis

E ) Senile ptosis

A

The correct response is Option E.

Senile ptosis is characterized by dehiscence of the levator aponeurosis, the most common cause of ptosis in the elderly. Elevation of the supratarsal crease is seen. Levator plication or advancement is required for correction.

Congenital ptosis is seen in young patients who have moderate-to-severe ptosis with absence of the eyelid crease and poor levator function. Upper eyelid frontalis sling is sometimes required for correction.

Facial nerve injury results in brow ptosis but not blepharoptosis.

Horner syndrome is marked by ptosis, miosis, and anhidrosis. It occurs after loss of sympathetic innervations of the superior cervical ganglion. The eyelid crease is usually not affected.

Myasthenia gravis may cause unilateral or bilateral ptosis that is exaggerated with fatigue. The condition typically affects young women or elderly men. Pharmacologic testing establishes the diagnosis.

43
Q

A 45-year-old woman comes to the office for consultation regarding lower eyelid blepharoplasty. Physical examination shows prominent eyes; exophthalmos is suspected. Measurement of the distance between the anterior border of the globe and the most anterior point of which of the following aspects of the orbital rim is most appropriate to confirm this diagnosis?

A ) Inferior

B ) Lateral

C ) Medial

D ) Radix

E ) Superior

A

The correct response is Option B.

Orbital morphology is increasingly recognized as an important predictor of postoperative complications in eyelid surgery. Several studies have shown increased lower eyelid morbidity in patients with exophthalmos. A recent study of 269 consecutive patients suggests that patients with enophthalmic orbits, combined with horizontal lower eyelid laxity, are also at increased risk. The Hertel exophthalmometer measures the distance between the anterior border of the globe and the most anterior point of the lateral aspect of the orbital rim. Enophthalmos is defined as less than 14 mm; midrange 15 to 18 mm, and exophthalmos greater than 18 mm.

44
Q

Which of the following anatomical events occurs during eyelid closure?
A ) The lacrimal canaliculi occlude

B ) The lacrimal diaphragm returns to a relaxed position

C ) The lacrimal puncta close

D ) The lacrimal sac collapses

E ) The nasolacrimal duct shortens

A

The correct response is Option C.

During eyelid closure, the lacrimal puncta close because of simple forced position. In contrast, during eyelid opening, the lacrimal puncta are open and in contact with the lacrimal lake at the medial aspect of the lower eyelid. The lacrimal sac is collapsed and empty at this stage, and the canaliculi are patent. Reopening the eyelid allows the lacrimal diaphragm to return to its resting position through cessation of the sphincter action of the orbicular muscle of the eye.

Upon eyelid closure, tears are milked lateral to medial. The deep heads of the preseptal muscles contract, shortening the canaliculi and closing their ampullae. Simultaneously, the deep heads of the preseptal muscles that are attached to the fascia of the sac (lacrimal diaphragm) pull the sac laterally. This creates negative pressure and results in the opening of the sac.

As the eyelids reopen, the lacrimal diaphragm returns to its relaxed position, creating sufficient pressure to propel the tears into the nasolacrimal duct. The canaliculi reopen at this phase to allow collection of more tears.

45
Q

A 65-year-old man has involutional ptosis of the left upper eyelid. Which of the following is the most important factor to consider when determining the appropriate management?

A ) Amount of brow ptosis

B ) Amount of excess eyelid skin

C ) Degree of levator function

D ) Distance between the tarsal edge and the brow

E ) Position of the supratarsal fold

A

The correct response is Option C.

Involutional ptosis, the most common type of acquired eyelid ptosis, is caused by a defect in the levator aponeurosis that occurs with stretching and allows downward positioning of the tarsal plate. Involutional ptosis is associated with good-to-excellent levator function, a high supratarsal crease, and thinning of the eyelid tissues above the tarsal plate.

The degree of levator function determines the required degree of correction. With a poor levator function (0 €“6 mm of excursion), frontalis suspension is required. With a moderate function (6 €“10 mm), levator resection (shortening) is required. For patients with excellent levator function (> 10 mm), aponeurotic surgery is appropriate.

The amount of brow ptosis will help determine whether additional corrective procedures are necessary. The amount of excess skin and the distance between the tarsal edge and the brow indicate the amount of excess skin to be excised in a blepharoplasty.

The position of the supratarsal fold indicates the cause of ptosis and is useful in determining placement of corrective incision.

46
Q

A 68-year-old woman is scheduled to undergo a bilateral blepharoplasty with unilateral repair of a levator aponeurosis dehiscence. During the dissection, the distal end of the dehisced levator muscle is reattached to the tarsal plate. Which of the following anatomic structures is most likely to be visualized deep to the repair?

A ) Capsulopalpebral fascia

B ) Conjunctiva

C ) Müller muscle

D ) Orbital septum

E ) Whitnall ligament

A

The correct response is Option C.

The layers of the eyelid are the conjunctiva, Müller muscle, levator muscle, orbital fat, orbital septum, retro-orbicularis oculi fat, orbicularis oculi muscle, and skin. The Müller muscle lies below the levator insertion to the superior border of the tarsus and is visualized during the levator muscle repair.

The conjunctiva is located deep to the Müller muscle and should not be visualized during the muscle reinsertion to the tarsus. The orbital septum and retro-orbicularis oculi are anterior and superior to the levator tendon. The Whitnall ligament is also superior to the levator tendon. The capsulopalpebral fascia is the extension of the lower eyelid retractors and fuses at the inferior aspect of the lower eyelid tarsus.

47
Q

A 40-year-old woman is scheduled to undergo surgical correction of bilateral upper eyelid dermatochalasis and mild blepharoptosis. A Fasanella-Servat procedure is planned. Which of the following is the most likely disadvantage of performing this procedure instead of a levator aponeurosis reinsertion?

A ) Dry eyes

B ) Eyelid asymmetry

C ) Incomplete correction of eyelid ptosis

D ) No removal of excess eyelid fold skin

E ) Overcorrection of eyelid position

A

The correct response is Option D.

Both the Fasanella-Servat procedure and the levator aponeurosis reinsertion can accomplish good surgical results and are accurate methods for the correction of mild to moderate ptosis. However, one of the major disadvantages of the Fasanella-Servat procedure is that it does not address excess skin of the eyelid fold.

Dry eyes, incomplete correction, overcorrection, and eyelid asymmetry are possible complications of both procedures.

48
Q

A 64-year-old man is evaluated for reconstruction of a defect of the lower eyelid following resection of a 1.4-cm nodular basal cell carcinoma. Examination shows an 80% full-thickness defect of the lateral lower eyelid. Which of the following is the most appropriate method of reconstruction?

A ) Cantholysis, lateral canthotomy, and primary closure

B ) Cheek advancement flap with composite graft for lining

C ) Composite graft from the ear

D ) Forehead flap with septal cartilage grafting

E ) Hughes tarsoconjunctival flap with skin grafting

A

The correct response is Option B.

This major (greater than 75%) defect of the lower eyelid has been categorized as a zone II defect by Spinelli and Jelks. The most appropriate method of reconstruction is the cheek advancement flap with a nasal septal cartilage and lining graft for internal lining. The cheek advancement flap can be elevated widely and rotated without tension to provide anterior coverage of the defect, while the composite graft is used for lining and support of the lower eyelid. Cantholysis, lateral canthotomy, and primary closure are most useful for defects that are less than 50%. Similarly, the Hughes tarsoconjunctival flap is best used in defects that are less than 50%, as larger flaps would result in significant deformity of the upper eyelid. Composite grafts from the ear are rarely used for lower eyelid reconstruction because the tissues are usually thicker than the lower eyelid and may be associated with partial or complete graft loss. Forehead flaps can be used for medial eyelid or canthal defects but are tertiary options in reconstruction of lateral lower eyelid defects.

49
Q

A 69-year-old woman comes to the office because she has had dryness, tearing, and irritation in both eyes for the past two years. Physical examination shows outward turning of the lower eyelid margin. Anterior eyelid distraction is 7 mm from the globe. Snap-back test is greater than one second. Muscle tone is normal bilaterally. Which of the following is the most likely cause of lower eyelid ectropion in this patient?

(A) Dehiscence of lower eyelid retractors

(B) Horizontal laxity of the lower eyelid

(C) Loss of lower eyelid tone secondary to paralysis of orbicularis oculi muscle

(D) Neoplasia within the lower eyelid causing the eyelid to be pulled away from the globe

(E) Vertical shortening of the anterior lamella of the eyelid

A

The correct response is Option B.

Causal factors leading to ectropion include horizontal laxity of the eyelid (involutional), vertical shortening of the anterior lamella of the eyelid (congenital or cicatricial), paralysis of the orbicularis oculi muscle (secondary to paralytic entropion) causing loss of eyelid muscular tone, and neoplasia within the lower eyelid pulling or forcing the eyelid away from the globe.

Physical examination findings of abnormal snap-back test and distraction test in the patient described suggest abnormal horizontal lid laxity.

50
Q

An 82-year-old woman comes to the office because she has had excessive tearing and irritation of both eyes for the past three months. Physical examination shows severe lower eyelid laxity, scleral show, downward drift of the lateral canthus, and a shortened intercanthal distance bilaterally. Entropion is suspected. Which of the following is most likely to confirm the suspected diagnosis?

(A) Animation test of the orbicularis oculi muscle

(B) Hertel exophthalmometry

(C) Jones dye test

(D) Lower eyelid snap-back test

(E) Schirmer test

A

The correct response is Option A.

Involutional entropion can be confused with involutional ectropion in the static state because of the associated lower eyelid laxity in both instances. The two conditions are completely distinguishable on animation of the orbicularis oculi muscle. Inversion of the lower eyelid occurs on attempted eyelid closure in cases of involutional entropion. Causal factors include orbicularis dysfunction with the preseptal portion overriding the pretarsal portion, upper eyelid €œclosure kick, € disinsertion of lower eyelid retractors, loss of horizontal and vertical eyelid support, and loss of orbital fat volume.

Hertel exophthalmometry is an objective measure of globe position in relation to the orbit; although enophthalmos can be associated with entropion, the test cannot distinguish it from ectropion. Jones dye test is used to assess the lacrimal drainage system. Lower eyelid snap-back test indicates laxity in both entropion and ectropion. The Schirmer test is an objective measure of lacrimal secretory capacity only.

51
Q

A 32 €‘year-old woman comes to the office for follow-up examination six months after she underwent repair of a fracture of the left orbit. A preoperative photograph is shown. Physical examination shows cicatricial ectropion of the lower eyelid. Which of the following is the most appropriate management?

(A) Complete tarsorrhaphy

(B) Pentagonal wedge resection of the lower eyelid

(C) Placement of a graft to the lower eyelid

(D) Punctal occlusion of the lower eyelid

(E) Resuspension of the lower eyelid retractors

A

The correct response is Option C.

Protection of the ocular surfaces is the primary goal in managing the cicatricial ectropion of the lower eyelid in the patient described. The patient €™s lower eyelid lacks height. Scarring between the capsulopalpebral fascia (lower eyelid retractors) and the orbital septum because of trauma or following aggressive lower eyelid blepharoplasty can cause vertical contracture of the lower eyelid, resulting in corneal exposure. Lower eyelid position can only be restored by placement of an autogenous or alloplastic spacer graft (hard palate, donor sclera, auricular cartilage, tarsus, and others), as in the postoperative image shown. In conjunction, eyelid-tightening procedures such as lateral canthopexy and canthoplasty address horizontal skin laxity.

Complete tarsorrhaphy is not an option for a young and active patient. It is an acceptable temporizing solution in cases of paralytic ectropion.

Pentagonal wedge resection of the lower eyelid is an adjunct procedure for cases of lower eyelid laxity with horizontal skin excess.

Punctal occlusion of the lower eyelid would alleviate the dryness caused by the increased exposure but would not restore normal anatomy of the lower eyelid in the patient described.

Repair of the lower eyelid retractors in conjunction with lateral canthal resuspension is used to correct lower eyelid entropion. The procedure would not address the scarring present in the patient described.

52
Q

For each patient with blepharoptosis, select the most likely diagnosis (A-D).

(A) Horner syndrome

(B) Injury to the facial (VII) nerve

(C) Myasthenia gravis

(D) Senile ptosis

88.

A 66 year old man with visual obstruction of the left eye and elevation of the supratarsal crease

89.

A 60 year old woman with general muscle fatigue and bilateral upper visual field obstruction that worsens at the end of the day

A

The correct response for Item 88 is D and for Item 89 is C.

Horner syndrome is marked by ptosis, miosis, and anhidrosis. It occurs after loss of sympathetic innervations of the superior cervical ganglion. The eyelid crease is usually not affected.

Injury to the facial (VII) nerve results in brow ptosis, not blepharoptosis.

Myasthenia gravis may cause unilateral or bilateral ptosis, which would be exaggerated by fatigue. The condition typically affects young women or elderly men. Pharmacologic testing establishes the diagnosis.

Senile ptosis is characterized by dehiscence of the levator aponeurosis, which is the most common cause of ptosis in elderly individuals. Elevation of the supratarsal crease is seen. Levator plication or advancement is required for correction.

53
Q

A 26-year old woman comes to the office after being referred by an ophthalmologist because of scleral show of the left lower eyelid. Three weeks ago, she sustained a laceration of the lower eyelid, which was sutured in the emergency department. Which of the following is the most appropriate next step in management?

(A) Tear supplementation and massage of the scar

(B) Injection of a corticosteroid into the scar and silicone sheeting

(C) Lysis of scar adhesions and septal cartilage interposition grafting

(D) Excision of the scar and full-thickness skin grafting

(E) Lateral canthopexy and cheek advancement flap

A

The correct response is Option A.

This patient has ectropion with eversion of the lower eyelid. In general, ectropion is classified as involutional, cicatricial, or neurogenic. The patient described has a cicatricial ectropion. At three weeks into the postoperative period, the wounds are still in the active phase of healing. Over the next six to nine months, as the collagen with the scar remodels, the lid may return to a more normal position. Therefore, during this time frame, the ectropion should be managed with scar tissue massage and a regimen to prevent complications related to dry eyes, such as tear supplementation and eye patching. Surgical intervention is warranted if there is no improvement after this time frame or if conservative therapy is ineffective and there is a risk of visual compromise.

Injection of a corticosteroid can reduce inflammation and soften the scar, allowing the eyelid to return to a better position. However, use of corticosteroids in the lower eyelid is not advocated because of the risks of tissue atrophy and skin discoloration.

Lysis of scar adhesions and the placement of an interpositional graft are indicated for cases of midlamellar scar. Placing upward traction on the eyelid can identify scar contracture in this location. If adequate skin is present, the lower eyelid should reach at least the level of the midpupil. If this movement is restricted, midlamellar scarring is likely.

Lateral canthopexy is indicated for lower eyelid malposition associated with lower eyelid laxity. Excision of the scar and skin grafting are indicated for ectropion secondary to cutaneous deficiency.

Excision of the scar and cheek advancement with a lateral canthopexy would not be a primary reconstruction procedure for this lesion.

54
Q

A 24-year-old man who works as a chef comes to the office because he has ectropion of the left lower eyelid and ulceration of the cornea five months after he sustained an oil burn to the left side of the face. On physical examination, the burn scars are hypertrophic and immature, extending from the infraorbital rim to the mandibular border. Which of the following is the most effective management of the ectropion?
(A) Full-thickness skin grafting
(B) Massage of the eyelid and injection of a corticosteroid
(C) Palatal mucosal grafting
(D) Tarsoconjunctival flap
(E) Tissue expansion

A

The correct response is Option A.

Cicatricial ectropion is best treated by full-thickness skin grafting to replace the skin deficit. A tarsal support procedure, such as a lateral canthoplasty, may be needed to fully restore the tone and position of the eyelid. Depending on their thickness and distribution, scars may be released by incision or excision. Cicatricial ectropion can be prevented by early surgical intervention with burn excision and grafting, thereby avoiding the need for late reconstructive procedures.

Most burn reconstruction should be delayed until scars have matured sufficiently, usually by one year after injury. Earlier intervention is needed when a vital skin function is impaired, such as corneal protection. Eyelid massage and corticosteroid injection are appropriate for minor scarring and scleral show after blepharoplasty but are unlikely to provide the rapid and complete relief needed for this patient.

Scarred skin does not expand effectively, limiting the role of tissue expansion in this patient. A tarsoconjunctival flap is useful for posterior lamella reconstruction of a full-thickness deficit of greater than 50% of the lower eyelid. The radial forearm free flap may replace a broad area of cheek injury, although the flap tissue is too bulky for effective eyelid skin replacement.

55
Q

Which of the following best differentiates the Asian upper eyelid from the Occidental upper eyelid?
(A) Absence of epicanthal folds
(B) Decreased amount of suborbicularis oculi fat
(C) More superior fusion of the orbital septum and levator aponeurosis
(D) Relative lack of insertions from the levator aponeurosis into the dermis
(E) Well-defined supratarsal lid fold with a larger pretarsal segment

A

The correct response is Option D.

The Asian eyelid has specific anatomic variations compared with the Occidental eyelid. First, in as much as 50% of the Asian population, there is a general lack of insertion of the levator aponeurosis into the dermis, causing a lack of a supratarsal fold. The fusion of the orbital septum to the levator aponeurosis is typically more caudad and decreases the width of the pretarsal segment of the supratarsal lid fold when it is present. There are generally increased amounts of retro-orbicularis oculi fat and suborbicularis oculi fat. The Asian eyelid is likely to have more epicanthal folds than is the Occidental eyelid.

56
Q

A 65-year-old man has a 3-cm-diameter open wound of the medial cheek inferior to the lower eyelid after undergoing Mohs’ micrographic surgery for excision of nodular basal cell carcinoma. Snap-back test of the lower eyelid shows poor tone. Reconstruction of the defect is performed with a cervicofacial flap. Which of the following is the most appropriate next step in management to avoid a deformity of the lower eyelid?
(A) Application of adhesive bandages to the lower eyelid and daily massage
(B) Placement of a temporary tarsorrhaphy (Frost) suture
(C) Reconstruction with a tarsoconjunctival flap
(D) Horizontal shortening of the lower eyelid and lateral canthopexy
(E) Full-thickness skin grafting of the lower eyelid

A

The correct response is Option D.

This patient is at increased risk for the development of ectropion of the lower eyelid, eversion of the lid margin away from the globe. The snap-back test can be used to assess horizontal laxity of the eyelid. After being pulled away from the eye, the time it takes for the lid to resume a normal position is measured. The result is graded from 0 (time indicating normal tone and laxity) to IV (time indicating loss of tone and severe laxity). Different lateral canthopexy and canthoplasty procedures have been described to prevent malposition of the lower eyelid and can be performed adjunctly with shortening of the lower eyelid. These procedures include repositioning of the inferior limb of the lateral retinaculum on the orbital rim; suturing of the lateral orbicularis oculi muscle to the orbital rim; suspension of a deepithelialized lower eyelid dermal pennant on the orbital rim; and suspension of a lateral orbit periosteal pennant on the orbital rim. All of the procedures commonly suspend or support the lateral canthus.

Ectropion of the lower eyelid may also result from vertical deficiency of the anterior lamella of the lid. Cicatricial changes with consequent vertical lid shortening may result from skin disease, trauma, and previous surgical procedures. Local therapy such as pressure and massage may be used to treat mild cicatricial changes. Further therapy to augment the anterior lamella with a full-thickness skin graft may also be required.

The use of tarsorrhaphy is a temporary means to protect the cornea from exposure. The suture itself will not prevent lower eyelid malposition. The tarsoconjunctival flap, which is elevated from the upper eyelid, may be used to reconstruct defects of the lower eyelid. In this case, there is no such defect.

57
Q

A 3-year-old boy is brought to the office by his parents because of new onset of bilateral epiphora. The boy’s parents say that similar symptoms occurred in one of their older children but resolved without treatment. On physical examination, the lashes of both lower eyelids rub against the inferior cornea. Which of the following is the pathophysiologic mechanism underlying this patient’s condition?
(A) Abnormal attachment of the canthal tendons
(B) Abnormal attachment of the orbital septum
(C) Enophthalmos
(D) Laxity of the tarsal plate
(E) Redundancy of skin and orbicularis muscle

A

The correct response is Option E.

The elevation of these tissues near the eyelid margin forces an upward and inward rotation of the lower lashes. A common result of this rotation is contact between the lower eyelashes and the cornea or inferior bulbar conjunctiva. Laxity of the tarsal plate may be seen as an atrophic change in adults with involutional entropion. Laxity of the canthal tendons is an involutional change and would not be expected in children. Abnormal septum attachments may occur as a postoperative or posttraumatic complication but are unlikely the cause of eyelash malposition.

58
Q

A 13-month-old girl has had tearing and discharge from the right eye since birth. Which of the following is the most appropriate management?
(A) Observation
(B) Instruction of the parents in massage with antibiotic ointment
(C) Silastic intubation
(D) Probing of the nasolacrimal duct
(E) Dacryocystorhinostomy

A

The correct response is Option D.

A child with congenital tearing is likely to have a nasolacrimal duct problem. Punctual agenesis, lacrimal sac fistula, and other rare abnormalities should be ruled out with dye disappearance testing, which usually is markedly asymmetric in a nasolacrimal duct problem. Generally, a nasolacrimal duct problem should be treated with massage and antibiotic drops until the child is age 12 to 13 months. For about 70% of children with tearing at age 6 months, this conservative treatment leads to resolution by age 12 months. If tearing persists, probing of the nasolacrimal duct should be performed. The longer probing is delayed beyond age 13 months, the greater the number and complexity of the procedures needed to successfully treat congenital dacryostenosis. Therefore, initial probing and irrigation should be performed before age 13 months.

Observation alone is not appropriate because it delays treatment, which increases the number and complexity of the procedures required. Massage with antibiotic ointment is not appropriate for this 13-month-old girl, although it could have been done before age 12 months. If probing is unsuccessful, Silastic intubation should be done. Dacryocystorhinostomy is reserved for those rare cases that do not respond to Silastic intubation.

Typically, the Jones test is not needed to make the diagnosis. The Jones I test involves instillation of fluorescein dye into the conjunctival sac. A cotton pledget is placed inside the nose close to the orifice of the nasolacrimal duct. Staining of the pledget, a positive test result, indicates that flow through the lacrimal system is uninhibited. A negative result on the Jones I test indicates obstruction but does not localize it in the upper or lower system. In the Jones II test, the punctum is anesthetized and dye is inserted through an irrigation cannula. Then the system is irrigated with saline. If dye is collected from the nose, a positive test result, the nasolacrimal duct is partially obstructed, but the upper system from the conjunctiva to the lacrimal sac is not obstructed.

59
Q

A 16-year-old girl has persistent ptosis of the eyelid (shown) six months after undergoing reconstruction of the forehead and supraorbital bar for fibrous dysplasia. One month postoperatively, a wound developed from extrusion of hardware through the medial eyelid. Debridement of the wound with excision of the involved inflammatory tissue and reconstruction with a pericranial flap were performed at that time. On current examination, no levator function is noted medially in the eyelid and lateral movement of the eyelid is minimal. Which of the following interventions is the most appropriate next step in management?
(A) Lysis of adhesions
(B) Kuhnt-Szymanowski procedure
(C) Fasanella-Servat procedure
(D) Suspension to the frontalis muscle with fascia lata grafting
(E) Advancement of the levator muscle

A

The correct response is Option D.

This patient has traumatic ptosis as a complication of prior surgical procedures. At the time of debridement, either a portion of the levator muscle was excised or the muscle function is limited by scar tissue. In either case, the function as shown in the photographs is minimal, and the ptosis is best managed by a frontalis suspension procedure. It is generally agreed the fascia lata is the material of choice for the sling. Alloplastic materials, such as silicone slings, can also be used but have a risk of extrusion and infection. The advantage of alloplastic sling reconstruction is that there are no risks to the donor site.

The Fasanella-Servat procedure involves excision of a portion of the conjunctiva, tarsus, orbital septum, levator aponeurosis, and Müller muscle. It can be used in cases of mild ptosis (1-2 mm). Levator function must be present.

The Kuhnt-Szymanowski procedure is used to correct lower eyelid ectropion. It involves a wedge excision of the lower eyelid.

Levator advancement surgery could possibly be used in this case if, during exploration of the eyelid, the levator was noted to be present and not compromised by scar. However, in this case, the levator mechanism was likely excised in the process of debriding the wound tract and inflammatory tissue. Therefore, in light of the degree of ptosis, the noted limited elevation of the upper eyelid, and the mechanism of injury, frontalis suspension is a better choice for ptosis repair in this patient.

With no evidence of muscle function, it is unlikely that adhesions alone are the cause of restricted eyelid movement. Therefore, lysing the scar would likely be inadequate treatment.

60
Q

A 2-year-old boy is brought to the office by his parents for evaluation of ptosis of the upper eyelids. On examination, there are no abnormalities of the right upper eyelid. Examination of the left eye shows 3.5 mm of ptosis of the upper lid, absence of the eyelid crease, and 3 mm of levator muscle function. Which of the following is the most appropriate management of the left eye?

(A) Observation
(B) Application of a patch
(C) Frontalis sling, upper eyelid
(D) Levator resection/advancement, upper eyelid
(E) Resection of Müller’s muscle, upper eyelid

A

The correct response is Option C.

The patient has 3.5 mm of ptosis of the left upper eyelid, which partially obstructs vision. Absence of the eyelid crease and 3 mm of levator function are highly suggestive of congenital ptosis. A frontalis sling of the left upper eyelid can correct these abnormalities by resetting the position of the affected eyelid.

Observation is not appropriate because the partial obstruction of vision could lead to amblyopia in the left eye. Application of a patch is inappropriate because it would deprive the left eye of all visual input, exacerbating the problem. Resection of Müller’s muscle from the left upper eyelid is an acceptable treatment of a small amount of ptosis (such as 1 mm) in an adult but would not correct this severe ptosis. Levator resection and advancement of the upper eyelid are the most common method of treating levator dehiscence in adults but would not be appropriate to treat a child with minimal levator function.

61
Q

A 57-year-old woman comes to the office for evaluation of drooping of the eyelids. Physical examination shows ptosis of the upper eyelids, high eyelid crease, margin to reflex distance (MRD) of 0 mm, and excellent levator function. Which of the following techniques is most appropriate for surgical correction of this patient=s abnormality?

(A) Fasanella-Servat
(B) Plication of the levator muscle
(C) Reanastomosis of the dehisced levator aponeurosis
(D) Resection of the orbicularis muscle
(E) Unilateral frontalis suspension using autogenous fascia lata

A

The correct response is Option C.

This patient has classic signs of blepharoptosis secondary to acquired dehiscence of the levator aponeurosis, such as high eyelid crease, excellent levator function, and an MRD of 0 mm. The MRD quantifies the distance between the lid margin and the pupillary reflex. To correct this blepharoptosis in the most anatomically correct fashion, surgery should be done to reanastomose the dehisced end of the levator aponeurosis to the superior aspect of the tarsus.

The other surgical options are used for different indications. A Fasanella-Servat procedure takes a posterior conjunctival approach to correct mild ptosis without levator disinsertion. Plication of the levator muscle alone does not provide long-lasting results and is a valid option only if the distal end of the muscle is not dehisced. Resection of the orbicularis muscle will not correct the ptosis. Unilateral frontalis suspension is the procedure of choice for patients with absent levator function and severe ptosis.

62
Q

A 6-year-old child who has had chronic bilateral epiphora since birth has been treated with corneal lubrication for the past year. Slit-lamp examination by the child’s pediatric ophthalmologist one week ago showed bilateral corneal staining. On physical examination, the lashes on both lower eyelids rub against the inferior cornea (shown above). Which of the following is the most appropriate management?

(A) Daytime taping of the margin of the lower lid
(B) Initiate nighttime lubrication of the eyes
(C) Lateral tarsal strip with repositioning of the eyelid margin
(D) Reinsertion of the retractors at the base of the tarsus and subtotal excision of the preseptal orbicularis
(E) Resection of redundant pretarsal skin and orbicularis muscle

A

The correct response is Option E.

The most common cause of epiblepharon is excess pretarsal skin and orbicularis oculi muscle at the lower eyelid margin. In this congenital anomaly, a fold of skin and underlying orbicularis muscle override the eyelid margin, often pushing the cilia against the globe. The eyelid margin and tarsus are stable and maintain the proper orientation. Epiblepharon usually affects the lower eyelids, is more common among Asians, and may be accentuated on downward gaze. Most cases resolve with facial growth during childhood. Surgical correction is needed when the lashes cause significant corneal injury. Epiblepharon requires resection of the redundant pretarsal skin and orbicularis muscle as well as placement of sutures between the tarsal plate and the subcutaneous tissue to create adhesions.

Taping of the lower eyelid is not practical in a child. Nighttime lubrication of the eyes is not sufficient treatment because the lashes continue to cause damage during the day, which may cause permanent corneal scarring. Lateral tarsal strip and repositioning of the eyelid margin are a common treatment for ectropion. Reinsertion of the retractors at the base of the tarsus and subtotal excision of the preseptal orbicularis are the treatment of choice for involutional entropion with horizontal laxity from the tarsus, vertical laxity due to attenuation or disinsertion of the lower eyelid retractors or orbital septum, and migration of the preseptal orbicularis in the pretarsal position.

63
Q

A 44-year-old woman has the defect shown above three days after undergoing excision of a squamous cell carcinoma from the lower eyelid. The resultant defect of the lower lid is 60%. Which of the following reconstructions is the most appropriate management?

(A) Composite contralateral lower lid graft
(B) Cutler-Beard flap
(C) Direct closure with cantholysis
(D) Hughes tarsoconjunctival flap
(E) Skin graft

A

The correct response is Option D.

This defect requires prompt surgical correction, because allowing the wound to granulate would result in a severely dysfunctional eyelid. With such a defect, a Hughes tarsoconjunctival flap is best used for reconstruction. This reconstruction involves taking a tarsoconjunctival flap from the upper eyelid, leaving 3 to 4 mm of the upper tarsus intact, and advancing the remaining tarsus and conjunctiva to the lower eyelid defect. This provides the posterior lamella of the eyelid. The anterior lamella is reconstructed by advancement of a skin-muscle flap from the lower eyelid or cheek.

Direct closure with cantholysis is appropriate for defects of less than 25% of the eyelid. The Cutler-Beard flap, which uses skin and muscle from the lower eyelid, is inappropriate for a patient with a lower eyelid defect. Because this patient=s defect affects more than 50% of the lower eyelid and because the eyelid requires a supportive base, a skin graft or composite contralateral lower eyelid graft would not provide adequate support.

64
Q

Hyphema results from traumatic hemorrhage of which of the following ocular structures?

(A) Anterior chamber
(B) Conjunctiva
(C) Lens
(D) Posterior chamber
(E) Vitreous chamber

A

The correct response is Option A.

Hyphema is traumatic hemorrhage of the anterior chamber of the eye, typically resulting from blunt trauma to ocular structures. The anterior chamber of the eye is bordered by the cornea anteriorly and the iris and central portion of the lens posteriorly. This structure is filled with aqueous humor originating from the ciliary processes in the posterior chamber and flowing through the pupil into the anterior chamber. In patients with hyphema, vessels are torn in the iris or in the ciliary body, leading to onset of hemorrhage. The blood collects in the most inferior section of the anterior chamber, obscuring the lower portion of the iris.

Significant hyphema may result in increased ocular pressure and/or permanent staining of the cornea. Because of the potential for these complications, screening is recommended in patients with facial trauma, and ophthalmologic referral is indicated in any patient with positive findings. Treatment involves administration of acetazolamide and corticosteroid eye drops to decrease ocular tension.

Bleeding that occurs in the bulbar or palpebral conjunctiva is referred to as subconjunctival hemorrhage. This condition is seen in patients with facial trauma and results from extravasation of conjunctival capillaries. It also occurs in association with zygomatic fractures that extend through the lateral orbital wall in which there is bleeding along the side of the orbit and into the subconjunctival interstitium. Treatment of the fracture is likely to resolve the hemorrhage.

The lens is an avascular structure that does not retain blood. Instead, it may subluxate or dislocate and cause a premature cataract.

The posterior chamber is located behind the iris and anterior to the suspensory ligament; it appears as a halo around the lens. Although this chamber adjoins the ciliary processes, which may hemorrhage, the blood is not retained within it, but instead flows into the anterior chamber.

The vitreous chamber is bordered by the lens and suspensory ligaments anteriorly and the retina posteriorly. It contains a dense, jelly-like fluid that maintains slight pressure and provides even contact of the retina against the choroid. In patients with retinal hemorrhages, the blood typically remains within the retinal tissue. Debris or blood that is released into the vitreous body is consumed by phagocytic cells or remains as vitreal floaters within the chamber.

65
Q

The common canaliculus enters the lacrimal sac at a point posterior to which of the following structures?

(A) Deep head of the preseptal muscle
(B) Deep head of the pretarsal muscle
(C) Lacrimal crest
(D) Medial canthal tendon
(E) Medial horn of the levator muscle

A

The correct response is Option D.

The common canaliculus enters the lacrimal sac at a point posterior to the medial canthal tendon. The medial canthal tendon is formed from the superficial heads of the pretarsal muscles and originates anterior to and above the lacrimal crest. The anterior and posterior lacrimal crests border the lacrimal fossa, which contains the lacrimal sac. The crests lie posterior to the canalicular entrance to the sac.

The deep head of the preseptal muscle originates from the posterior lacrimal crest, just above the deep heads of the pretarsal muscles. The deep heads of the pretarsal muscles extend posterior to the lacrimal sac and join with the diaphragm of the scar to originate immediately behind the posterior lacrimal crest. The medial horn of the levator muscle lies superior to the medial canthal tendon.

66
Q

A 25-year-old man has excessive tear secretion and a chronic mucocele of the lacrimal sac. Jones I dye testing shows no dye, while Jones II dye testing shows dye within the tear sac. Dilatation of the puncta with probing and irrigation does not restore the patency of the lacrimal system. In order to restore nasolacrimal drainage in this patient, which of the following is the most appropriate operative procedure?

(A) Insertion of a Jones tube
(B) Canaliculodacryocystorhinostomy
(C) Conjunctivodacryocystostomy
(D) Conjunctivorhinostomy
(E) Dacryocystorhinostomy

A

The correct response is Option E.

Jones dye testing is performed initially in this patient who has excessive tear secretion. The Jones I dye test involves the instillation of 2% fluorescein dye into the conjunctival fornices. Recovery of the dye (a positive test) indicates that flow through the lacrimal system is uninhibited. Negative findings on the Jones I test are indicative of functional obstruction; if this occurs, the Jones II dye test should be performed immediately. With this test, the nasolacrimal system is irrigated with 1 mL of saline via an irrigation cannula. If dye-stained fluid is found at the inferior turbinate, there is partial obstruction of the lower canalicular system, most likely at the nasolacrimal duct. If there is dye within the tear sac, obstruction of the nasolacrimal duct can be diagnosed; the canaliculus and lacrimal pump are unaffected. If no dye-stained fluid is found in the nose (a negative test), the obstruction is most likely at the canalicular level.
In patients with positive Jones II dye tests, dilation of the puncta should then be performed; any fluid that passes into the nose indicates that the obstruction of the nasolacrimal duct has been cleared, and further probing is unnecessary. However, probing and irrigation do not restore patency in this situation. Therefore, this patient has negative findings on the Jones I dye test but positive findings on the Jones II dye test, indicating a partial obstruction of the nasolacrimal duct. In this patient who has duct obstruction resulting from a chronic mucocele of the lacrimal sac, dacryocystorhinostomy is indicated.

Insertion of a Jones tube is indicated in patients who have complete obstruction of the lacrimal puncta.

Canaliculodacryocystorhinostomy, with intubation, is recommended for management of strictures found at the junction of the common canaliculus and the lacrimal sac, as well as for obstructions at the level of the canaliculus.

Although conjunctivodacryocystostomy is appropriate for patients who have obstruction at the canalicular level, this technique is associated with disruption of the lacrimal sac, resulting in unreliable long-term patency.

Conjunctivorhinostomy is indicated in patients who have absence or obliteration of the tear sac.

67
Q

A 42-year-old woman has drooping of the left eyelid two weeks after undergoing upper eyelid blepharoplasty and injection of botulinum toxin into the forehead. Physical examination shows 3 mm of ptosis of the left eyelid and 13 mm of levator excursion. When the left eyelid is closed voluntarily, the iris shadow can be visualized through the eyelid. The left tarsal crease is elevated 3 mm when compared with the right tarsal crease.

Which of the following types of ptosis is the most likely cause of these findings?

(A) Botulinum toxin-induced
(B) Traumatic aponeurotic
(C) Traumatic mechanical
(D) Traumatic myogenic
(E) Traumatic neurogenic

A

The correct response is Option B.

This patient has traumatic aponeurotic ptosis, which is characterized by the physical examination findings of good levator function, elevation of the eyelid crease, and the ability to visualize the shadow of the iris with eyelid closure (positive Nesi sign). In this type of ptosis, the levator aponeurosis is detached from the tarsal plate. Early reattachment of the levator is most likely to produce the best result.

Patients with botulinum toxin-induced ptosis will have a more prominent decrease in levator function. In patients with mechanical, myogenic, or neurogenic ptosis, the shadow of the iris cannot be visualized through the eyelid.

68
Q

A 68-year-old woman desires rejuvenation of the eyelids. Physical examination shows excessive skin of the lower eyelids and 2 mm of scleral show bilaterally. Which of the following is the most appropriate management?

(A) Injection of botulinum toxin (Botox) into the lateral canthal region
(B) Carbon dioxide laser resurfacing of the lower eyelids
(C) Lower eyelid blepharoplasty with excision of skin
(D) Lower eyelid blepharoplasty with lateral canthopexy
(E) Transconjunctival blepharoplasty

A

The correct response is Option D.

This 68-year-old woman has substantially diminished elasticity of the skin of the lower eyelids. Because of this, she is at increased risk for development of ectropion postoperatively if blepharoplasty with skin excision is performed alone and the laxity of the lower eyelid skin is not corrected. Therefore, the most appropriate management is lower eyelid blepharoplasty combined with lateral canthopexy, which will decrease the laxity of the lower eyelids and lower the risk for ectropion.

Injection of botulinum toxin will not rejuvenate the lower eyelids, and carbon dioxide laser resurfacing of the lower eyelids will only increase the risk for ectropion if canthopexy is not performed concomitantly. Transconjunctival blepharoplasty alone will not correct the excessive skin of the lower eyelids.

69
Q

Three months after undergoing bilateral lower eyelid blepharoplasty, a 45-year-old woman has scleral show, round, sad-looking eyes, and symptoms of ocular irritation, including photophobia and excessive tearing. The most likely cause of these findings is scarring between the orbital septum and which of the following structures?

(A) Capsulopalpebral fascia
(B) Inferior oblique muscle
(C) Orbicularis oculi muscle
(D) Orbital rim
(E) Tarsal plate

A

The correct response is Option A.

The findings seen in this patient are consistent with retraction of the lower eyelids, a common complication following surgery of the lower eyelids and midface. Lower eyelid retraction is defined as inferior malpositioning of the lower eyelid margin without eyelid eversion. Affected patients have round, sad-looking eyes, scleral show, and symptoms of ocular irritation, including photophobia, excessive tearing, and nocturnal lagophthalmos. Ocular lubricants typically provide only minimal relief.

Potential causes of lower eyelid retraction include scarring between the orbital septum and capsulopalpebral fascia, laxity of the lateral canthal tendon, and descent of the structures of the midface. Appropriate management is based on patient findings, which may include relaxation of the tarsal margin, skin shortening, and/or deep lamellar scar contracture.

70
Q

A 30-year-old woman is undergoing examination six months after sustaining periorbital lacerations in a motor vehicle collision. She has 3.5 mm of ptosis, and levator function is greater than 10 mm. Which of the following is the most appropriate management?

(A) Advancement of MŸeller’s muscle
(B) Eyebrow suspension
(C) Fasanella-Servat procedure
(D) Repositioning of the levator aponeurosis
(E) Levator resection

A

The correct response is Option D.

This patient who has ptosis of 3.5 mm with good levator function (greater than 10 mm) is best managed with repositioning of the levator aponeurosis. In patients who develop ptosis following an operative procedure or trauma, multiple etiologic mechanisms, including neurologic, myogenic, and mechanical, may be the cause. Because injuries to the levator complex may be concealed by acute edema, the surgeon should allow for resolution of the edema, myoneural recovery, and scar softening. After this has occurred, the degree of ptosis and the extent of levator function can be measured in order to determine the appropriate surgical procedure.

Advancement of MŸeller’s muscle will not correct 3.5 mm of ptosis. Eyebrow suspension is most appropriate for patients who have poor levator function (less than 4 mm) and greater than 3 mm of ptosis. The Fasanella-Servat procedure shortens the lower components of the eyelid (ie, the tarsus, conjunctiva, and MŸller’s muscle). The modification of this procedure spares the muscle and is best for patients who have levator function of greater than 10 mm and ptosis of less than 2 mm. Levator resection is reserved for those patients who have levator function between 4 mm and 10 mm, and ptosis of greater than 3 mm.

71
Q

A 6-year-old boy has eyelid ptosis. Examination shows 4 mm of ptosis and 2 mm of levator excursion. Facial nerve function is normal. Which of the following is the most appropriate management?

(A) Blepharoplasty
(B) Frontalis suspension
(C) Placement of gold weights in the eyelids
(D) Levator plication
(E) Levator resection

A

The correct response is Option B.

Frontalis suspension is the most appropriate procedure for correction of the poor levator function and severe ptosis seen in this 6-year-old boy. Appropriate preoperative evaluation should be performed in any patient with ptosis to classify the type and severity of the ptosis and the amount of levator function. Ptosis is classified as congenital or acquired. Patients with congenital ptosis often have poor function of the levator muscle, characterized by absence of eyelid excursion and a static eyebrow. In acquired ptosis, levator function is typically moderate to good. The amount of levator function is critical to determine the appropriate surgical correction.

Blepharoplasty is appropriate in patients with excess skin and fat around the eyelids, but not in patients with true ptosis. Placement of gold weights in the eyelids is indicated for patients with facial paralysis to assist with eyelid closure. Plication and/or resection of the levator muscle is appropriate for patients who have moderate ptosis and levator function that is rated as fair to good.

72
Q

What is the approximate percentage of patients undergoing reconstruction of fractures of the orbital floor through a transconjunctival approach who will develop ectropion?

(A) 0%
(B) 10%
(C) 20%
(D) 30%
(E) 40%

A

The correct response is Option A.

In one study of 80 patients undergoing reconstruction of orbital blowout fractures and fractures of the zygomaticomaxillary complex via a preseptal transconjunctival approach, ectropion was not identified as a complication. Only one case of transient entropion was identified, and only 2% of patients in this study reported any complications. Another study of 35 patients who were undergoing secondary orbital procedures reported that 4% of these patients subsequently developed ectropion. In a third study of 400 patients, one patient was identified as having ectropion. In contrast, the rate of ectropion in patients undergoing the traditional subciliary approach to reconstruction has been shown to be as high as 25%.

73
Q

A 62-year-old woman has visual obstruction of the right eye. On examination, she has ptosis of 3 to 4 mm of the right upper eyelid and an elevated supratarsal crease. These findings are most consistent with which of the following conditions?

(A) Dehiscence of the levator aponeurosis
(B) Facial nerve injury
(C) Horner’s syndrome
(D) Myasthenia gravis
(E) Periorbital fat atrophy

A

The correct response is Option A.

The findings in this patient are most consistent with dehiscence of the levator aponeurosis, which is the most common cause of ptosis in elderly persons. Attenuation of the levator aponeurosis typically results. Levator advancement is performed for correction.

Facial nerve injury is a sequela of trauma and may be characterized by eyebrow ptosis.

Horner’s syndrome occurs as a result of sympathetic denervation of the superior cervical ganglion. It is characterized by ptosis, myosis, and anhidrosis. The eyelid creases and levator muscle are typically unaffected.

Unilateral or bilateral ptosis secondary to myasthenia gravis is exacerbated with fatigue and can present in young women and elderly men. Neostigmine testing is used to establish the diagnosis.

Periorbital fat atrophy results in pseudoptosis and enophthalmos of the globe.

74
Q

The photograph shown above is of a 56-year-old man who underwent open reduction and internal fixation of a malar complex fracture on the right and cranial bone grafting of the right orbital floor three months ago after sustaining bony injuries in a motor vehicle collision. He had no skin lacerations at the time of injury.

Which of the following is the most likely cause of the lower eyelid deformity?

(A) Entrapment of Lockwood’s ligament
(B) Inferior displacement of the orbital floor
(C) Loss of skin elasticity
(D) Periorbital fat atrophy
(E) Shortening of the posterior lamella

A

The correct response is Option E.

This patient’s lower eyelid deformity is most likely caused by shortening of the posterior lamella. The lower eyelid is formed by the anterior, middle, and posterior lamellae. The anterior lamella consists of skin and orbicularis oculi muscle. The orbital septum comprises the middle lamella. The posterior lamella, or capsulopalpebral fascia, is comprised of the tarsus muscle, lower lid retractors, and conjunctiva. Injury or scarring of any of these structures can result in malpositioning of the lower eyelid, as seen in this patient. Shortening and scarring of the posterior lamella and septum are most common.

Entrapment of Lockwood’s ligament would lead to a loss of globe support, and inferior displacement of the cranial bone grafts and orbital floor would result in dystopia. Scleral show and ectropion resulting from excess skin excision are more typical of cosmetic blepharoplasty than internal fixation. Periorbital fat atrophy can result in scleral show and a change in globe position but rarely causes ectropion in patients with traumatic orbital injuries.

75
Q

A 45-year-old woman has had severe epiphora on the right side for the past four months. She sustained a comminuted naso-orbital ethmoid fracture when she was struck in the face by a softball six months ago; open reduction and internal fixation were performed immediately after injury. Dacryocystography shows obstruction of the nasolacrimal duct.

Which of the following is the most appropriate operative management?

(A) Conjunctivodacryocystostomy
(B) Conjunctivodacryocystorhinostomy
(C) Conjunctivorhinostomy
(D) Dacryocystorhinostomy
(E) Dacryocystostomy

A

The correct response is Option D.
This patient has developed nasolacrimal duct obstruction as a complication following open reduction and internal fixation of a comminuted naso-orbital ethmoid fracture. The level of obstruction must be determined in order to correctly bypass the stricture or damaged portion of the lacrimal system. This can be accomplished by various methods, including canalicular injection and/or intubation, fluorescein staining of the eye, and radiologic testing.
Dacryocystorhinostomy is used for correction of nasolacrimal duct obstruction. Many methods of dacryocystorhinostomy have been described. The single lacrimal flap technique, as well as other techniques that do not involve flaps, has produced long-term patency rates of 90%.

Conjunctivodacryocystostomy and conjunctivodacryocystorhinostomy are procedures used for reconstruction in a patient who has obstruction at the canalicular level. Conjunctivorhinostomy is used in patients who have absence or obliteration of the tear sac. Dacryocystostomy involves intubation of the tear sac, which would not be beneficial in this patient.

76
Q

The patient shown in the photograph above will be at increased risk for development of which of the following complications following four-eyelid blepharoplasty? ** no picture**

(A) Diplopia
(B) Dry eye syndrome
(C) Entropion
(D) Hematoma
(E) Ptosis

A

The correct response is Option B.

The patient shown in the photograph has minimal exophthalmos and moderate scleral show. Such clinical findings, as well as proptosis, hypotonia of the lower eyelids, and maxillary hypoplasia, are significant predictors of dry eye syndrome, while low tear film is less predictive of dry eye syndrome. Therefore, appropriately cautious management is critical in patients who have these anatomic findings and are considering blepharoplasty. Surgery can still be performed with the necessary modifications and adequate ocular protection. One study reported that 65% of patients who developed dry eye syndrome following blepharoplasty had normal findings on preoperative Schirmer’s testing.
This patient would not be at risk for diplopia, which is more closely related to edema, hematoma, and wound infection. Entropion occurs as a result of damage to the ciliary margins of the eyelid resulting from an excessively close incision (leading to angulation of the upper border of the tarsal plate) or spasm of the upper portion of the orbicularis oculi muscle. However, this patient’s findings are not indicative of postoperative entropion. Development of hematoma is rarely predictable. The findings are not consistent with ptosis, which can be worsened postoperatively if it is not identified before the procedure.

77
Q

The photographs shown above are of a 58-year-old man who has recurrent painless edema of the eyelids. Three upper eyelid blepharoplasty procedures over the past 30 years have not resolved this condition. On physical examination, the skin of the upper eyelids is thin, and results of snap testing are poor. ** no picture**

These findings are most consistent with which of the following?

(A) Blepharochalasis
(B) Dermatochalasis
(C) Dry eye syndrome
(D) Pachydermoperiostosis
(E) Senile ptosis

A

The correct response is Option A.

This 58-year-old man has findings consistent with blepharochalasis, a condition of unknown cause that results from a developmental deficiency of elastic tissue within the eyelids. It most often occurs during early adulthood. Patients with blepharochalasis, or dermatopysis palpebrum, have recurrent episodes of mild to moderate, painless edema of the eyelids. Over time, the episodes become more frequent, resulting in a permanent “baggy” appearance of the eyelids with thinning and wrinkling of the eyelid skin. Herniation of orbital fat occurs, leading to an exacerbation of symptoms. Excision of redundant tissue is indicated for management of visual obstruction.

Patients with dermatochalasis have occasional episodes of visual obstruction resulting from excess eyelid skin. Dry eye syndrome is caused by corneal exposure following blepharoplasty and manifests as pain, dryness, and blurred vision. Pachydermoperiostosis, or idiopathic hypertrophic osteoarthropathy, is a familial condition of unknown cause characterized by progressive enlargement of the eyelids, hands, feet, and toes. The conjunctivae are covered by hypertrophic papillae. Ptosis and visual obstruction are common. Senile ptosis results from progressive attenuation of the levator aponeurosis. Levator advancement is appropriate management.

78
Q

A 43-year-old woman has miosis, anhidrosis, and blepharoptosis measuring 2 mm. On examination, the eyelid crease is normal and function of the levator muscle is good. Which of the following is the most likely diagnosis?

(A) Blepharophimosis syndrome
(B) Congenital ptosis
(C) Horner’s syndrome
(D) Involutional ptosis
(E) Myasthenia gravis

A

The correct response is Option C.

This 43-year-old woman has Horner’s syndrome, which is caused by sympathetic denervation of the superior cervical ganglion. Typical findings include ptosis, miosis, and anhidrosis. The eyelid creases and levator muscle are typically unaffected.

Blepharophimosis syndrome is a congenital condition consisting of ptosis, telecanthus, and phimosis of the upper eyelid fissure.

Congenital ptosis is a developmental dystrophy that affects the levator muscle. In patients with congenital ptosis, eyelid creases are poorly defined and levator function is poor. These patients are at increased risk for the development of strabismus and amblyopia.

Involutional ptosis is the most common type of acquired ptosis. This condition results from progressive thinning of the levator aponeurosis and subsequent downward shifting of the tarsal plate. The function of the levator muscle is good despite its progressive thinning. The eyelid creases are typically raised.

Patients with ptosis due to myasthenia gravis frequently have unilateral or bilateral ptosis that is exacerbated with fatigue. This disorder is most frequent in young women and elderly men. Neostigmine testing is used to establish the diagnosis.

79
Q

A 40-year-old woman has steady, lancinating pain in the globe and orbit and episodes of vomiting six hours after undergoing blepharoplasty of the lower eyelids. She says that she sees sparkles and flashes and has the sensation similar to a “window shade” closing over the lower half of her range of vision.

These findings are most consistent with which of the following?

(A) Acute glaucoma
(B) Adverse effects of anesthesia
(C) Migraine
(D) Retrobulbar hematoma
(E) Transient ischemic attack

A

The correct response is Option D.

This patient has findings consistent with retrobulbar hematoma, a complication of blepharoplasty that, if untreated, can result in loss of vision. Retrobulbar hematoma is most frequently characterized by steady, severe, lancinating pain in the globe and orbit (mimicking symptoms of acute glaucoma), which can occur alone or with scintillating scotomas (ie, sparkles and flashes, mimicking the symptoms of severe migraine) and hemianopsia or amaurosis fugax (ie, findings similar to a “window shade” pulled over the lower half of the visual field, mimicking a transient ischemic attack). Other symptoms associated with the development of hematoma following blepharoplasty include early discharge from the eye, perioperative and postoperative vomiting, and coughing. The use of aspirin-containing products has also been associated. A positive finding on Valsalva’s maneuver may be diagnostic.

Appropriate management includes surgical exploration and lateral canthotomy, with ophthalmologic consultation. Mannitol and carbonic anhydrase inhibitors can also be administered to decrease intraocular pressure and reestablish blood flow.

Although eye pain following surgery may result from abrasion of the cornea during anesthesia, scotomas and loss of vision would not be associated.

80
Q

After undergoing repair of an orbital fracture, a patient has progressive loss of vision resulting from the development of a retrobulbar hematoma. In order to immediately relieve intraocular pressure, which of the following structures should be released?

(A) Lateral canthal tendon
(B) Levator aponeurosis
(C) Orbicularis oculi muscle
(D) Tarsal plate
(E) Tenon’s capsule

A

The correct response is Option A.

Intraocular pressure is similar to compartment syndrome in that an excessive amount of pressure on the optic nerve resulting from bleeding or edema can increase intraorbital pressure and ultimately lead to blindness. An acute increase in intraorbital pressure can occur following orbital fracture repair, blepharoplasty, or other surgical procedures. Physical examination in an affected patient will show visual loss associated with a pupillary defect, such as a loss of pupillary reaction to light. Increased intraocular pressure can be measured at the bedside using a tonometer. Lateral canthotomy, or release of the lateral canthal tendon, should be performed immediately to relieve pressure and restore sight. Any proptosis that is present will be accentuated immediately after surgery and can thus be diagnosed at this time; it can be measured to confirm adequate release of the check-rein ocular support. Surgical exploration can also be performed after lateral canthotomy to control any persistent bleeding.
In contrast, eyelid structures such as the levator aponeurosis, orbicularis oculi muscle, and tarsal plate do not need to be released. Because these structures lie external to the support sling created by the lateral canthal tendon and Lockwood’s suspensory ligament, they already have at least partial mobility.

Tenon’s capsule covers the globe and extraocular muscles, creating a barrier between these structures and the orbital fat. The lower portion of the capsule comprises Lockwood’s suspensory ligament. Because of their position proximal to the globe, it is more prudent to detach the lateral canthal tendon from the bony rim in order to allow for advancement of the intraorbital contents. Surgical detachment of the lateral canthal tendon will disrupt the globe only minimally. Any loss of lower eyelid height can be restored later with lateral canthopexy.

81
Q

A 48-year-old man has traumatic telecanthus after sustaining a naso-orbito-ethmoid fracture. Open reduction and internal fixation are performed with transnasal wiring of the fracture fragments surrounding the medial canthal tendon. At follow-up examination one year after the initial injury, the patient has epiphora of the right eye. Findings on Jones I dye testing are negative and Jones II dye testing are positive.

Which of the following anatomic sites is the most likely origin of this patient’s findings?

(A) Canaliculi
(B) Nasolacrimal duct
(C) Lacrimal gland
(D) Punctum
(E) Superior meatus

A

The correct response is Option B.

The Jones I dye test involves the instillation of fluorescein dye into the conjunctival sac. A cotton pledget is placed inside the nose close to the orifice of the nasolacrimal duct; staining of the pledget, also known as a positive test, indicates that flow through the lacrimal system is uninhibited. In patients who have both a positive Jones I test and epiphora, hypersecretion of the lacrimal gland is the most likely cause. Negative findings on the Jones I test are indicative of obstruction; however, the test does not localize the obstruction to the upper or lower system.

With the Jones II dye test, the punctum is anesthetized and dilated, and an irrigation cannula is inserted; the system is then irrigated with saline. If dye-stained fluid is collected from the nose, which indicates a positive test, there is partial obstruction of the nasolacrimal duct; the upper system, from the conjunctiva to the lacrimal sac, is not obstructed. If no dye-stained fluid is found in the nose (a negative test), but there is reflux through the opposite canaliculus, obstruction of the lower system, and not the upper system, can be diagnosed. In contrast, if findings on the Jones II test are negative, and there is reflux through the irrigated canaliculus, the obstruction can be localized to the upper system. Therefore, a patient who has negative findings on the Jones I dye test but positive findings on the Jones II dye test has a partial obstruction of the nasolacrimal duct.

The nasolacrimal duct empties into the inferior meatus; in contrast, the posterior ethmoid air cells empty into the superior meatus and are not connected to the lacrimal system.

82
Q

A 2-year-old child is being evaluated because he has deformities of the eyelids and upper face. Examination shows large epicanthal folds of the lower eyelids with epicanthus inversus, horizontal shortening of the eyelids, and 5 mm of ptosis bilaterally. Levator excursion is 4 mm.

These findings are most consistent with

(A) blepharochalasis
(B) blepharophimosis syndrome
(C) blepharospasm
(D) congenital epicanthus
(E) epiblepharon

A

The correct response is Option B.

Blepharophimosis syndrome, which is classified according to three types, is a form of congenital ptosis. Patients with blepharophimosis syndrome, type 1 have large epicanthal folds with epicanthus inversus, horizontally shortened eyelids, and severe ptosis. Patients with type 2 have telecanthus, absence of the epicanthal folds, severe bilateral ptosis, absence of levator function, and skin shortage involving all four lids. Blepharophimosis syndrome, type 3 involves absence of epicanthal folds, telecanthus, an antimongoloid slant of the palpebral fissures, severe ptosis, mild orbital hypertelorism, and skin deficiencies. As with many congenital syndromes, the malformations are isolated. Forehead and ear anomalies are also common. Although epicanthus and epiblepharon comprise a portion of this patient’s condition, the presence of severe congenital ptosis leads to a diagnosis of blepharophimosis syndrome type 1.

Surgical correction of blepharophimosis syndrome involves repair of the epicanthal folds and correction of eyelid ptosis. Levator resection, medial canthoplasty, and fascial suspension techniques have all been used in combination to correct the abnormalities. A five-flap technique that combines double Z-plasties and a Y-to-V flap is most often used for repair of the epicanthal folds. Ideally, the canthus should lie halfway between the pupil and the center of the nasal bridge following all repair procedures.

Blepharochalasis and blepharospasm are not seen in patients with blepharophimosis syndrome.

83
Q

A 68-year-old woman seeks correction of drooping eyelids and impaired upward gaze. Physical examination shows excessive hooding of the upper eyelid skin; visual field testing confirms obstruction in the upper fields. Levator excursion is 14 mm bilaterally. There is 2 mm of ptosis of the left eyelid; the right eye is unaffected.

In addition to blepharoplasty, which of the following is the most appropriate management?

(A) Division of MŸller’s muscle
(B) Fasanella-Servat procedure
(C) Fascial sling
(D) Levator advancement
(E) Resection of the levator muscle

A

The correct response is Option D.

Bilateral blepharoplasty with fat pad removal and ptosis repair using levator advancement will address this woman’s visual field defect and mild ptosis. These procedures are used for patients with normal levator function (defined as greater than 10 mm). Bilateral upper eyelid blepharoplasty alone or in conjunction with fat pad removal would not correct the ptosis, while repair of the ptosis only would not address the visual field obstruction.

This patient has a common problem that requires thorough preoperative evaluation. Visual field obstruction is evaluated clinically and verified using standard visual field testing. Examination of levator function involves stabilizing the brow and measuring the excursion of the upper eyelid margin from downward gaze to upward gaze with the eyes fixed on a distant point. The normal distance between the upper and lower limbi across the pupil is 11 mm. The upper limbus should rest 2 mm below the superior edge of the iris and 2 mm above the superior edge of the pupil.

Division of Muller’s muscle would not correct the ptosis.

The Fasanella-Servat procedure is used to correct minimal ptosis but is a more difficult, complicated procedure than levator plication. Accessibility to involved structures is limited with this procedure.

Bilateral blepharoplasty combined with fat pad removal and ptosis repair using a fascial sling is recommended to correct congenital ptosis, defined as ptosis of more than 4 mm and levator function of less than 5 mm.

Resection of the levator muscle is excessive and unnecessary in patients with minimal acquired ptosis.

84
Q

The photographs shown above are of a 58-year-old man who has recurrent painless edema of the eyelids. Three upper eyelid blepharoplasty procedures over the past 30 years have not resolved this condition. On physical examination, the skin of the upper eyelids is thin, and results of snap testing are poor.

These findings are most consistent with which of the following?

(A) Blepharochalasis
(B) Dermatochalasis
(C) Dry eye syndrome
(D) Pachydermoperiostosis
(E) Senile ptosis

A

The correct response is Option A.

This 58-year-old man has findings consistent with blepharochalasis, a condition of unknown cause that results from a developmental deficiency of elastic tissue within the eyelids. It most often occurs during early adulthood. Patients with blepharochalasis, or dermatopysis palpebrum, have recurrent episodes of mild to moderate, painless edema of the eyelids. Over time, the episodes become more frequent, resulting in a permanent “baggy” appearance of the eyelids with thinning and wrinkling of the eyelid skin. Herniation of orbital fat occurs, leading to an exacerbation of symptoms. Excision of redundant tissue is indicated for management of visual obstruction.

Patients with dermatochalasis have occasional episodes of visual obstruction resulting from excess eyelid skin. Dry eye syndrome is caused by corneal exposure following blepharoplasty and manifests as pain, dryness, and blurred vision. Pachydermoperiostosis, or idiopathic hypertrophic osteoarthropathy, is a familial condition of unknown cause characterized by progressive enlargement of the eyelids, hands, feet, and toes. The conjunctivae are covered by hypertrophic papillae. Ptosis and visual obstruction are common. Senile ptosis results from progressive attenuation of the levator aponeurosis. Levator advancement is appropriate management.

85
Q

A 50-year-old woman has right eyelid ptosis of 2 mm two days after undergoing uncomplicated four-eyelid blepharoplasty under local anesthesia. On physical examination, there is moderate edema of the upper and lower eyelids.

Which of the following is the most appropriate next step in management?

(A) Reassurance and continued follow-up examinations
(B) Eyelid massage and stretching exercises
(C) Administration of phenylephrine eyedrops
(D) Immediate operative exploration of the eyelid
(E) Levator plication seven days after the initial procedure

A

The correct response is Option A.

Ptosis of a mild to moderate degree is a common finding following blepharoplasty; common causes include postoperative edema of the eyelids and hemorrhage into MŸller’s muscle. Because these complications generally resolve spontaneously over time, reassurance and observation with frequent follow-up examinations are most appropriate.

Eyelid massage and stretching exercises are effective for management of early ectropion of the lower lids. Phenylephrine eyedrops are only indicated if the ptosis is caused by Horner’s syndrome. Although operative exploration is warranted in patients who have eyelid discrepancies following blepharoplasty, it would not be useful in this patient who did not initially undergo repair of the levator mechanism. Instead, surgical treatment in this patient should be delayed for two to six months to allow for spontaneous recovery.

86
Q

A 25-year-old man has ectropion and excessive scleral show one year after sustaining a chemical burn of the lower right eyelid, which was allowed to heal without surgical intervention. He currently uses ocular ointments daily. Which of the following is the most appropriate management?

(A) Scar massage and intralesional injection of a corticosteroid
(B) Full-thickness skin grafting
(C) Insertion of a gold eyelid weight
(D) Lateral canthoplasty
(E) Lateral tarsal wedge excision

A

The correct response is Option B.

Ectropion involves eversion of the eyelid margin; it frequently occurs in the lower eyelid as a result of the pull of gravity on the unsupported eyelid tissue. It is usually the result of mechanical (involutional or senile), cicatricial, or neurogenic causes.

This patient has classic cicatricial ectropion, which has resulted from abnormally healing burn wounds. The ectropion has occurred as a result of scar contracture of the anterior lamella of the lower eyelid, leading to excessive scleral show and exposure keratopathy. Full-thickness skin grafting is recommended to replace lost tissue and prevent secondary contracture. In addition, complete release of contracted soft tissues and use of added supporting materials such as cartilage should be considered.

Scar massage and intralesional injection of a corticosteroid would not improve a fixed, foreshortened lower eyelid.

Neurogenic ectropion is best treated by correction of the associated upper eyelid lagophthalmos using inserted prosthetic devices (eg, gold eyelid weights).

In patients with involutional ectropion, there is progressive laxity of the lower eyelid; the lower eyelid retractors or capsulopalpebral fascia becomes disinserted from the inferior border of the tarsal plate. Corrective surgical procedures include lateral canthoplasty, lateral wedge excision, and the Kuhnt-Szymanowski technique, which involves excision of a full-thickness wedge from the region of the lateral canthus. The skin excision is then hidden under a subciliary incision.

87
Q

A 70-year-old woman has a skin defect with a diameter of 1 cm after undergoing resection of a basal cell carcinoma of the right upper eyelid. Primary closure of the defect is not possible. Which of the following is most appropriate for cutaneous full-thickness coverage of the defect?

(A) Cross-lid flap
(B) Retroauricular skin graft
(C) Skin graft from the contralateral upper eyelid
(D) Supraclavicular skin graft
(E) Wedge resection and primary closure of the remaining eyelid

A

The correct response is Option C.

A skin graft from the contralateral upper eyelid should be used for full-thickness coverage of this patient’s defect. Because older patients typically have dermatochalasia, there is often sufficient skin in the unaffected upper eyelid that can be harvested and used to cover small skin deficits.

A cross-lid flap is more appropriate for coverage of a full-thickness defect. Grafts harvested from the retroauricular and supraclavicular regions are better used for skin grafting of the face due to their optimal color match; however, this skin is often too thick for use in the upper eyelid. Wedge resection is an excessive, unnecessary procedure.

88
Q

Which of the following bones comprises the greatest portion of the medial orbital wall?

(A) Ethmoid
(B) Lacrimal
(C) Maxilla
(D) Palatine
(E) Sphenoid

A

The correct response is Option A.

The medial orbital wall is comprised primarily of the orbital plate of the ethmoid bone. This bone is made up of a horizontal or cribriform plate, a midline perpendicular plate that forms the nasal septum, and symmetric lateral masses. The outer wall of each lateral mass is the medial orbital wall, the inner walls are the sidewalls of the nasal
Knowledge of the anatomy of the medial orbital wall is important when diagnosing and treating extensive orbital blowout fractures, which often include the orbital floor. These fractures can extend to involve the inferomedial hillock and central section of the medial orbital wall; if this occurs, the orbital contents may be displaced into the maxillary and ethmoid sinuses. If the anatomic volume of the bony orbit is not restored surgically, the patient may develop posttraumatic enophthalmos and diplopia. This will most likely occur in those patients who have an increase in bony orbital volume of greater than 5%.

The lacrimal and palatine bones give off smaller contributions to the medial orbital wall, as does the lesser wing of the sphenoid bone. The orbital floor is comprised of the maxilla medially and the zygoma anteriorly.

89
Q

A 30-year-old woman is undergoing examination six months after sustaining periorbital lacerations in a motor vehicle collision. She has 3.5 mm of ptosis, and levator function is greater than 10 mm. Which of the following is the most appropriate management?

(A) Advancement of MŸeller’s muscle
(B) Eyebrow suspension
(C) Fasanella-Servat procedure
(D) Repositioning of the levator aponeurosis
(E) Levator resection

A

The correct response is Option D.

This patient who has ptosis of 3.5 mm with good levator function (greater than 10 mm) is best managed with repositioning of the levator aponeurosis. In patients who develop ptosis following an operative procedure or trauma, multiple etiologic mechanisms, including neurologic, myogenic, and mechanical, may be the cause. Because injuries to the levator complex may be concealed by acute edema, the surgeon should allow for resolution of the edema, myoneural recovery, and scar softening. After this has occurred, the degree of ptosis and the extent of levator function can be measured in order to determine the appropriate surgical procedure.

Advancement of MŸeller’s muscle will not correct 3.5 mm of ptosis. Eyebrow suspension is most appropriate for patients who have poor levator function (less than 4 mm) and greater than 3 mm of ptosis. The Fasanella-Servat procedure shortens the lower components of the eyelid (ie, the tarsus, conjunctiva, and MŸller’s muscle). The modification of this procedure spares the muscle and is best for patients who have levator function of greater than 10 mm and ptosis of less than 2 mm. Levator resection is reserved for those patients who have levator function between 4 mm and 10 mm, and ptosis of greater than 3 mm.

90
Q

A 6-year-old boy has eyelid ptosis. Examination shows 4 mm of ptosis and 2 mm of levator excursion. Facial nerve function is normal. Which of the following is the most appropriate management?

(A) Blepharoplasty
(B) Frontalis suspension
(C) Placement of gold weights in the eyelids
(D) Levator plication
(E) Levator resection

A

The correct response is Option B.

Frontalis suspension is the most appropriate procedure for correction of the poor levator function and severe ptosis seen in this 6-year-old boy. Appropriate preoperative evaluation should be performed in any patient with ptosis to classify the type and severity of the ptosis and the amount of levator function. Ptosis is classified as congenital or acquired. Patients with congenital ptosis often have poor function of the levator muscle, characterized by absence of eyelid excursion and a static eyebrow. In acquired ptosis, levator function is typically moderate to good. The amount of levator function is critical to determine the appropriate surgical correction.

Blepharoplasty is appropriate in patients with excess skin and fat around the eyelids, but not in patients with true ptosis. Placement of gold weights in the eyelids is indicated for patients with facial paralysis to assist with eyelid closure. Plication and/or resection of the levator muscle is appropriate for patients who have moderate ptosis and levator function that is rated as fair to good.

91
Q

A 25-year-old man has ectropion of the left lower eyelid and excessive tearing of the eye one month after undergoing reduction of an orbital floor fracture through a subciliary approach. The conjunctiva does not appear to be markedly irritated. Which of the following is the most appropriate management?

(A) Observation with massage and taping of the eyelid
(B) Injection of a corticosteroid
(C) Lateral canthopexy
(D) Scar release and grafting of the conjunctiva using a mucosa graft
(E) Skin-muscle blepharoplasty of the lower eyelid

A

The correct response is Option A.

In this patient who has ectropion of the left lower eyelid and excessive tearing of the left eye one month after undergoing fracture repair, the most appropriate management is observation with massage and taping of the eyelid. Scleral show and ectropion of the lower eyelid occur frequently in patients who have undergone surgery involving the lower eyelid. These complications typically improve over time. At four weeks, collagen is not compact; it will remodel significantly over the following two to six weeks. Operative procedures, such as lateral canthopexy or lamellar release and grafting, are indicated only if there is permanent scarring.

Corticosteroid injections are associated with unnecessary risks, such as orbital puncture and tissue thinning, and are not indicated when improvement is to be expected. Blepharoplasty procedures would only worsen the ectropion.

92
Q

What is the approximate percentage of patients undergoing reconstruction of fractures of the orbital floor through a transconjunctival approach who will develop ectropion?

(A) 0%
(B) 10%
(C) 20%
(D) 30%
(E) 40%

A

The correct response is Option A.

In one study of 80 patients undergoing reconstruction of orbital blowout fractures and fractures of the zygomaticomaxillary complex via a preseptal transconjunctival approach, ectropion was not identified as a complication. Only one case of transient entropion was identified, and only 2% of patients in this study reported any complications. Another study of 35 patients who were undergoing secondary orbital procedures reported that 4% of these patients subsequently developed ectropion. In a third study of 400 patients, one patient was identified as having ectropion. In contrast, the rate of ectropion in patients undergoing the traditional subciliary approach to reconstruction has been shown to be as high as 25%.

93
Q

Which of the following bones comprises the greatest portion of the medial orbital wall?

(A) Ethmoid
(B) Lacrimal
(C) Maxilla
(D) Palatine
(E) Sphenoid

A

The correct response is Option A.

The medial orbital wall is comprised primarily of the orbital plate of the ethmoid bone. This bone is made up of a horizontal or cribriform plate, a midline perpendicular plate that forms the nasal septum, and symmetric lateral masses. The outer wall of each lateral mass is the medial orbital wall, the inner walls are the sidewalls of the nasal
Knowledge of the anatomy of the medial orbital wall is important when diagnosing and treating extensive orbital blowout fractures, which often include the orbital floor. These fractures can extend to involve the inferomedial hillock and central section of the medial orbital wall; if this occurs, the orbital contents may be displaced into the maxillary and ethmoid sinuses. If the anatomic volume of the bony orbit is not restored surgically, the patient may develop posttraumatic enophthalmos and diplopia. This will most likely occur in those patients who have an increase in bony orbital volume of greater than 5%.

The lacrimal and palatine bones give off smaller contributions to the medial orbital wall, as does the lesser wing of the sphenoid bone. The orbital floor is comprised of the maxilla medially and the zygoma anteriorly.

94
Q

A 62-year-old woman has visual obstruction of the right eye. On examination, she has ptosis of 3 to 4 mm of the right upper eyelid and an elevated supratarsal crease. These findings are most consistent with which of the following conditions?

(A) Dehiscence of the levator aponeurosis
(B) Facial nerve injury
(C) Horner’s syndrome
(D) Myasthenia gravis
(E) Periorbital fat atrophy

A

The correct response is Option A.

The findings in this patient are most consistent with dehiscence of the levator aponeurosis, which is the most common cause of ptosis in elderly persons. Attenuation of the levator aponeurosis typically results. Levator advancement is performed for correction.

Facial nerve injury is a sequela of trauma and may be characterized by eyebrow ptosis.

Horner’s syndrome occurs as a result of sympathetic denervation of the superior cervical ganglion. It is characterized by ptosis, myosis, and anhidrosis. The eyelid creases and levator muscle are typically unaffected.

Unilateral or bilateral ptosis secondary to myasthenia gravis is exacerbated with fatigue and can present in young women and elderly men. Neostigmine testing is used to establish the diagnosis.

Periorbital fat atrophy results in pseudoptosis and enophthalmos of the globe.

95
Q

The photograph shown above is of a 56-year-old man who underwent open reduction and internal fixation of a malar complex fracture on the right and cranial bone grafting of the right orbital floor three months ago after sustaining bony injuries in a motor vehicle collision. He had no skin lacerations at the time of injury. ** no picture**

Which of the following is the most likely cause of the lower eyelid deformity?

(A) Entrapment of Lockwood’s ligament
(B) Inferior displacement of the orbital floor
(C) Loss of skin elasticity
(D) Periorbital fat atrophy
(E) Shortening of the posterior lamella

A

The correct response is Option E.

This patient’s lower eyelid deformity is most likely caused by shortening of the posterior lamella. The lower eyelid is formed by the anterior, middle, and posterior lamellae. The anterior lamella consists of skin and orbicularis oculi muscle. The orbital septum comprises the middle lamella. The posterior lamella, or capsulopalpebral fascia, is comprised of the tarsus muscle, lower lid retractors, and conjunctiva. Injury or scarring of any of these structures can result in malpositioning of the lower eyelid, as seen in this patient. Shortening and scarring of the posterior lamella and septum are most common.

Entrapment of Lockwood’s ligament would lead to a loss of globe support, and inferior displacement of the cranial bone grafts and orbital floor would result in dystopia. Scleral show and ectropion resulting from excess skin excision are more typical of cosmetic blepharoplasty than internal fixation. Periorbital fat atrophy can result in scleral show and a change in globe position but rarely causes ectropion in patients with traumatic orbital injuries.

96
Q

The patient shown in the photograph above will be at increased risk for development of which of the following complications following four-eyelid blepharoplasty? ** no picture**

(A) Diplopia
(B) Dry eye syndrome
(C) Entropion
(D) Hematoma
(E) Ptosis

A

The correct response is Option B.

The patient shown in the photograph has minimal exophthalmos and moderate scleral show. Such clinical findings, as well as proptosis, hypotonia of the lower eyelids, and maxillary hypoplasia, are significant predictors of dry eye syndrome, while low tear film is less predictive of dry eye syndrome. Therefore, appropriately cautious management is critical in patients who have these anatomic findings and are considering blepharoplasty. Surgery can still be performed with the necessary modifications and adequate ocular protection. One study reported that 65% of patients who developed dry eye syndrome following blepharoplasty had normal findings on preoperative Schirmer’s testing.
This patient would not be at risk for diplopia, which is more closely related to edema, hematoma, and wound infection. Entropion occurs as a result of damage to the ciliary margins of the eyelid resulting from an excessively close incision (leading to angulation of the upper border of the tarsal plate) or spasm of the upper portion of the orbicularis oculi muscle. However, this patient’s findings are not indicative of postoperative entropion. Development of hematoma is rarely predictable. The findings are not consistent with ptosis, which can be worsened postoperatively if it is not identified before the procedure.

97
Q

A 45-year-old woman has had severe epiphora on the right side for the past four months. She sustained a comminuted naso-orbital ethmoid fracture when she was struck in the face by a softball six months ago; open reduction and internal fixation were performed immediately after injury. Dacryocystography shows obstruction of the nasolacrimal duct.

Which of the following is the most appropriate operative management?

(A) Conjunctivodacryocystostomy
(B) Conjunctivodacryocystorhinostomy
(C) Conjunctivorhinostomy
(D) Dacryocystorhinostomy
(E) Dacryocystostomy

A

The correct response is Option D.

This patient has developed nasolacrimal duct obstruction as a complication following open reduction and internal fixation of a comminuted naso-orbital ethmoid fracture. The level of obstruction must be determined in order to correctly bypass the stricture or damaged portion of the lacrimal system. This can be accomplished by various methods, including canalicular injection and/or intubation, fluorescein staining of the eye, and radiologic testing.
Dacryocystorhinostomy is used for correction of nasolacrimal duct obstruction. Many methods of dacryocystorhinostomy have been described. The single lacrimal flap technique, as well as other techniques that do not involve flaps, has produced long-term patency rates of 90%.

Conjunctivodacryocystostomy and conjunctivodacryocystorhinostomy are procedures used for reconstruction in a patient who has obstruction at the canalicular level. Conjunctivorhinostomy is used in patients who have absence or obliteration of the tear sac. Dacryocystostomy involves intubation of the tear sac, which would not be beneficial in this patient.

98
Q

A 45-year-old woman is being evaluated after undergoing upper eyelid blepharoplasty. Examination shows persistent fullness involving the lateral third of the upper eyelids. These findings are most consistent with which of the following?

(A) Descending lacrimal glands
(B) Eyebrow ptosis
(C) Lateral compartment fat
(D) Subcutaneous fat deposits
(E) Supraorbital bony prominences

A

The correct response is Option A.

In this patient who has undergone upper eyelid blepharoplasty, the persistent upper eyelid fullness is most likely a result of descending lacrimal glands. Appropriate management of this condition can include resuspension of the glands, which has been advocated by some surgeons. However, excision of the glands is not recommended because it may result in the development of keratoconjunctivitis sicca.

Eyebrow ptosis would most likely be seen over the entire eyebrow, not just the lateral third. Lateral fat compartments are found in the lower eyelids but not in the upper eyelids. Although subcutaneous fat deposits can cause lateral fullness of the eyelids, they typically pass beyond the region of the canthus. Bony prominences would most likely involve the entire eyebrow.

99
Q

A 25-year-old man has ectropion of the left lower eyelid and excessive tearing of the eye one month after undergoing reduction of an orbital floor fracture through a subciliary approach. The conjunctiva does not appear to be markedly irritated. Which of the following is the most appropriate management?

(A) Observation with massage and taping of the eyelid
(B) Injection of a corticosteroid
(C) Lateral canthopexy
(D) Scar release and grafting of the conjunctiva using a mucosa graft
(E) Skin-muscle blepharoplasty of the lower eyelid

A

The correct response is Option A.

In this patient who has ectropion of the left lower eyelid and excessive tearing of the left eye one month after undergoing fracture repair, the most appropriate management is observation with massage and taping of the eyelid. Scleral show and ectropion of the lower eyelid occur frequently in patients who have undergone surgery involving the lower eyelid. These complications typically improve over time. At four weeks, collagen is not compact; it will remodel significantly over the following two to six weeks. Operative procedures, such as lateral canthopexy or lamellar release and grafting, are indicated only if there is permanent scarring.

Corticosteroid injections are associated with unnecessary risks, such as orbital puncture and tissue thinning, and are not indicated when improvement is to be expected. Blepharoplasty procedures would only worsen the ectropion.

100
Q

After undergoing repair of an orbital fracture, a patient has progressive loss of vision resulting from the development of a retrobulbar hematoma. In order to immediately relieve intraocular pressure, which of the following structures should be released?

(A) Lateral canthal tendon
(B) Levator aponeurosis
(C) Orbicularis oculi muscle
(D) Tarsal plate
(E) Tenon’s capsule

A

The correct response is Option A.

Intraocular pressure is similar to compartment syndrome in that an excessive amount of pressure on the optic nerve resulting from bleeding or edema can increase intraorbital pressure and ultimately lead to blindness. An acute increase in intraorbital pressure can occur following orbital fracture repair, blepharoplasty, or other surgical procedures. Physical examination in an affected patient will show visual loss associated with a pupillary defect, such as a loss of pupillary reaction to light. Increased intraocular pressure can be measured at the bedside using a tonometer. Lateral canthotomy, or release of the lateral canthal tendon, should be performed immediately to relieve pressure and restore sight. Any proptosis that is present will be accentuated immediately after surgery and can thus be diagnosed at this time; it can be measured to confirm adequate release of the check-rein ocular support. Surgical exploration can also be performed after lateral canthotomy to control any persistent bleeding.
In contrast, eyelid structures such as the levator aponeurosis, orbicularis oculi muscle, and tarsal plate do not need to be released. Because these structures lie external to the support sling created by the lateral canthal tendon and Lockwood’s suspensory ligament, they already have at least partial mobility.

Tenon’s capsule covers the globe and extraocular muscles, creating a barrier between these structures and the orbital fat. The lower portion of the capsule comprises Lockwood’s suspensory ligament. Because of their position proximal to the globe, it is more prudent to detach the lateral canthal tendon from the bony rim in order to allow for advancement of the intraorbital contents. Surgical detachment of the lateral canthal tendon will disrupt the globe only minimally. Any loss of lower eyelid height can be restored later with lateral canthopexy.

101
Q
A